Docsity
Docsity

Prepare for your exams
Prepare for your exams

Study with the several resources on Docsity


Earn points to download
Earn points to download

Earn points by helping other students or get them with a premium plan


Guidelines and tips
Guidelines and tips

NCLEX RN ACTUAL EXAM TEST BANK OF REAL QUESTIONS & ANSWERS 2023/2024 100%Complete Graded A, Exams of Nursing

NCLEX RN ACTUAL EXAM TEST BANK OF REAL QUESTIONS & ANSWERS 2023/2024 100%Complete Graded A+ QUESTION 1 Which classification of drugs is contraindicated for the client with hypertrophic cardiomyopathy? A. Positive inotropes B. Vasodilators C. Diuretics D. Antidysrhythmics Answer: A Explanation: (A) Positive inotropic agents should not be administered owing to their action of increasing myocardial contractility. Increased ventricular contractility would increase outflow tract obstruction in the client with hypertrophic cardiomyopathy. (B) Vasodilators are not typically prescribed but are not contraindicated. (C) Diuretics are used with caution to avoid causing hypovolemic A. (D) Antidysrhythmics are typically needed to treat both atrial and ventricular dysrhythmias. QUESTION 2 Signs and symptoms of an allergy attack include which of the following? A. Wheezing on inspiration B. Increased respiratory rate C. Circumoral cyanosis D. Prolonged expiration Answer: D Explanation: (A) Wheezing oc

Typology: Exams

2023/2024

Available from 02/13/2024

HESISOLUTIONS
HESISOLUTIONS 🇬🇧

69 documents

1 / 545

Toggle sidebar

Related documents


Partial preview of the text

Download NCLEX RN ACTUAL EXAM TEST BANK OF REAL QUESTIONS & ANSWERS 2023/2024 100%Complete Graded A and more Exams Nursing in PDF only on Docsity! NCLEX RN ACTUAL EXAM TEST BANK OF REAL QUESTIONS & ANSWERS 2023/2024 100%Complete Graded A+ QUESTION 1 Which classification of drugs is contraindicated for the client with hypertrophic cardiomyopathy? A. Positive inotropes B. Vasodilators C. Diuretics D. Antidysrhythmics Answer: A Explanation: (A) Positive inotropic agents should not be administered owing to their action of increasing myocardial contractility. Increased ventricular contractility would increase outflow tract obstruction in the client with hypertrophic cardiomyopathy. (B) Vasodilators are not typically prescribed but are not contraindicated. (C) Diuretics are used with caution to avoid causing hypovolemic A. (D) Antidysrhythmics are typically needed to treat both atrial and ventricular dysrhythmias. QUESTION 2 Signs and symptoms of an allergy attack include which of the following? A. Wheezing on inspiration B. Increased respiratory rate C. Circumoral cyanosis D. Prolonged expiration Answer: D Explanation: (A) Wheezing occurs during expiration when air movement is impaired because of constricted edematous bronchial lumin A. (B) Respirations are difficult, but the rate is frequently normal. (C) The circumoral area is usually pale. Cyanosis is not an early sign of hypoxi A. (D) Expiration is prolonged because the alveoli are greatly distended and air trapping occurs. QUESTION 3 A client confides to the nurse that he tasted poison in his evening meal. This would be an example of what type of hallucination? A. Auditory B. Gustatory C. Olfactory D. Visceral Answer: B Explanation: (A) Auditory hallucinations involve sensory perceptions of hearing. (B) Gustatory hallucinations involve sensory perceptions of taste. (C) Olfactory hallucinations involve sensory perceptions of smell. (D) Visceral hallucinations involve sensory perceptions of sensation. NCLEX RN ACTUAL EXAM TEST BANK OF REAL QUESTIONS & ANSWERS 2023/2024 100%Complete Graded A+ QUESTION 4 Which of the following findings would be abnormal in a postpartal woman? A. Chills shortly after delivery B. Pulse rate of 60 bpm in morning on first postdelivery day C. Urinary output of 3000 mL on the second day after delivery D. An oral temperature of 101F (38.3C) on the third day after delivery Answer: D Explanation: (A) Frequently the mother experiences a shaking chill immediately after delivery, which is related to a nervous response or to vasomotor changes. If not followed by a fever, it is clinically innocuous. (B) The pulse rate during the immediate postpartal period may be low but presents no cause for alarm. The body attempts to adapt to the decreased pressures intra-abdominally as well as from the reduction of blood flow to the vascular bed. (C) Urinary output increases during the early postpartal period (12–24 hours) owing to diuresis. The kidneys must eliminate an estimated 2000–3000 mL of extracellular fluid associated with a normal pregnancy. (D) A temperature of 100.4F (38C) may occur after delivery as a result of exertion and dehydration of labor. However, any temperature greater than 100.4F needs further investigation to identify any infectious process. QUESTION 5 A six-month-old infant has been admitted to the emergency room with febrile seizures. In the teaching of the parents, the nurse states that: A. Sustained temperature elevation over 103F is generally related to febrile seizures B. Febrile seizures do not usually recur C. There is little risk of neurological deficit and mental retardation as sequelae to febrile seizures D. Febrile seizures are associated with diseases of the central nervous system Answer: C Explanation: (A) The temperature elevation related to febrile seizures generally exceeds 101F, and seizures occur during the temperature rise rather than after a prolonged elevation. (B) Febrile seizures may recur and are more likely to do so when the first seizure occurs in the 1st year of life. (C) There is little risk of neurological deficit, mental retardation, or altered behavior secondary to febrile seizures. (D) Febrile seizures are associated with disease of the central nervous system. QUESTION 6 A client diagnosed with bipolar disorder continues to be hyperactive and to lose weight. Which of the following nutritional interventions would be most therapeutic for him at this time? A. Small, frequent feedings of foods that can be carried B. Tube feedings with nutritional supplements C. Allowing him to eat when and what he wants D. Giving him a quiet place where he can sit down to eat meals Answer: A Explanation: (A) The manic client is unable to sit still long enough to eat an adequate meal. Small, frequent feedings with NCLEX RN ACTUAL EXAM TEST BANK OF REAL QUESTIONS & ANSWERS 2023/2024 100%Complete Graded A+ Answer: A Explanation: (A) The recommended range is 70–120 mg/dL to reduce the risk of perinatal mortality. (B, C, D) These levels are not recommended. The higher the blood glucose, the worse the prognosis for the fetus. Hypoglycemia can also have detrimental effects on the fetus. QUESTION 13 When evaluating a client with symptoms of shock, it is important for the nurse to differentiate between neurogenic and hypovolemic shock. The symptoms of neurogenic shock differ from hypovolemic shock in that: A. In neurogenic shock, the skin is warm and dry B. In hypovolemic shock, there is a bradycardia C. In hypovolemic shock, capillary refill is less than 2 seconds D. In neurogenic shock, there is delayed capillary refill Answer: A Explanation: (A) Neurogenic shock is caused by injury to the cervical region, which leads to loss of sympathetic control. This loss leads to vasodilation of the vascular beds, bradycardia resulting from the lack of sympathetic balance to parasympathetic stimuli from the vagus nerve, and the loss of the ability to sweat below the level of injury. In neurogenic shock, the client is hypotensive but bradycardiac with warm, dry skin. (B) In hypovolemic shock, the client ishypotensive and tachycardiac with cool skin. (C) In hypovolemic shock, the capillary refill would be>5 seconds. (D) In neurogenic shock, there is no capillary delay, the vascular beds are dilated, and peripheral flow is good. QUESTION 14 A 55-year-old man is admitted to the hospital with complaints of fatigue, jaundice, anorexia, and clay-colored stools. His admitting diagnosis is ―rule out hepatitis.‖ Laboratory studies reveal elevated liver enzymes and bilirubin. In obtaining his health history, the nurse should assess his potential for exposure to hepatitis. Which of the following represents a high-risk group for contracting this disease? A. Heterosexual males B. Oncology nurses C. American Indians D. Jehovah‘s Witnesses Answer: B Explanation: (A) Homosexual males, not heterosexual males, are at high risk for contracting hepatitis. (B) Oncology nurses are employed in high-risk areas and perform invasive procedures that expose them to potential sources of infection. (C) The literature does not support the idea that any ethnic groups are at higher risk. (D) There is no evidence that any religious groups are at higher risk. QUESTION 15 A schizophrenic client has made sexual overtures toward her physician on numerous occasions. During lunch, the client tells the nurse, ―My doctor is in love with me and wants to marry me.‖ This client is using which of the following defense mechanisms? NCLEX RN ACTUAL EXAM TEST BANK OF REAL QUESTIONS & ANSWERS 2023/2024 100%Complete Graded A+ A. Displacement B. Projection C. Reaction formation D. Suppression Answer: B Explanation: (A) Displacement involves transferring feelings to a more acceptable object. (B) Projection involves attributing one‘s thoughts or feelings to another person. (C) Reaction formation involves transforming an unacceptable impulse into the opposite behavior. (D) Suppression involves the intentional exclusion of unpleasant thoughts or experiences. QUESTION 16 When teaching a sex education class, the nurse identifies the most common STDs in the United States as: A. Chlamydia B. Herpes genitalis C. Syphilis D. Gonorrhea Answer: A Explanation: (A) Chlamydia trachomatis infection is the most common STD in the United States. The Centers for Disease Control and Prevention recommend screening of all high-risk women, such as adolescents and women with multiple sex partners. (B) Herpes simplex genitalia is estimated to be found in 5–20 million people in the United States and is rising in occurrence yearly. (C) Syphilis is a chronic infection caused by Treponema pallidum. Over the last several years the number of people infected has begun to increase. (D) Gonorrhea is a bacterial infection caused by the organism Neisseria gonorrhoeae. Although gonorrhea is common, chlamydia is still the most common STD. QUESTION 17 The nurse assists a client with advanced emphysema to the bathroom. The client becomes extremely short of breath while returning to bed. The nurse should: A. Increase his nasal O2 to 6 L/min B. Place him in a lateral Sims‘ position C. Encourage pursed-lip breathing D. Have him breathe into a paper bag Answer: C Explanation: (A) Giving too high a concentration of O2 to a client with em-physema may remove his stimulus to breathe. (B) The client should sit forward with his hands on his knees or an overbed table and with shoulders elevated. (C) Pursed-lip breathing helps the client to blow off CO2 and to keep air passages open. (D) Covering the face of a client extremely short of breath may cause anxiety and further increase dyspnea. NCLEX RN ACTUAL EXAM TEST BANK OF REAL QUESTIONS & ANSWERS 2023/2024 100%Complete Graded A+ QUESTION 18 In a client with chest trauma, the nurse needs to evaluate mediastinal position. This can best be done by: A. Auscultating bilateral breath sounds B. Palpating for presence of crepitus C. Palpating for trachial deviation D. Auscultating heart sounds Answer: C Explanation: (A) No change in the breath sounds occurs as a direct result of the mediastinal shift. (B) Crepitus can occur owing to the primary disorder, not to the mediastinal shift. (C) Mediastinal shift occurs primarily with tension pneumothorax, but it can occur with very large hemothorax or pneumothorax. Mediastinal shift causes trachial deviation and deviation of the heart‘s point of maximum impulse. (D) No change in the heart sounds occurs as a result of the mediastinal shift. QUESTION 19 Clinical manifestations seen in left-sided rather than in right-sided heart failure are: A. Elevated central venous pressure and peripheral edema B. Dyspnea and jaundice C. Hypotension and hepatomegaly D. Decreased peripheral perfusion and rales Answer: D Explanation: (A, B, C) Clinical manifestations of right-sided heart failure are weakness, peripheral edema, jugular venous distention, hepatomegaly, jaundice, and elevated central venous pressure. (D) Clinical manifestations of left- sided heart failure are left ventricular dysfunction, decreased cardiac output, hypotension, and the backward failure as a result of increased left atrium and pulmonary artery pressures, pulmonary edema, and rales. QUESTION 20 In assessing cardiovascular clients with progression of aortic stenosis, the nurse should be aware that there is typically: A. Decreased pulmonary blood flow and cyanosis B. Increased pressure in the pulmonary veins and pulmonary edema C. Systemic venous engorgement D. Increased left ventricular systolic pressures and hypertrophy Answer: D Explanation: (A) These signs are seen in pulmonic stenosis or in response to pulmonary congestion and edema and mitral stenosis. (B) These signs are seen primarily in mitral stenosis or as a late sign in aortic stenosis after left ventricular failure. (C) These signs are seen primarily in right-sided heart valve dysfunction. (D) Left ventricular hypertrophy occurs to increase muscle mass and overcome the stenosis; left ventricular pressures increase as left ventricular volume increases owing to insufficient emptying. NCLEX RN ACTUAL EXAM TEST BANK OF REAL QUESTIONS & ANSWERS 2023/2024 100%Complete Graded A+ extend the depth of burn are A. (B) The use of large amounts of water to flush the area is recommended for chemical burns. (C) Calcium chloride is not recommended therapy and would likely worsen the problem. (D) Lanolin is of no benefit in the initial treatment of a chemical injury and may actually extend a thermal injury. QUESTION 27 Dietary planning is an essential part of the diabetic client‘s regimen. The American Diabetes Association recommends which of the following caloric guidelines for daily meal planning? A. 50% complex carbohydrate, 20%–25% protein, 20%–25% fat B. 45% complex carbohydrate, 25%–30% protein, 30%–35% fat C. 70% complex carbohydrate, 20%–30% protein, 10%–20% fat D. 60% complex carbohydrate, 12%–15% protein, 20%–25% fat Answer: D Explanation: (A) The percentage of carbohydrates is too low to maintain blood sugar levels. The percent range of protein is too high and may cause extra workload on the kidney as it is metabolized. (B) The percentage of carbohydrates is too low to maintain blood sugar levels. The percent range of protein is too high and may cause extra workload on the kidney. (C) The percentage of carbohydrates is too high; the percent range of protein is too high, and of fat, too low. (D) This combination provides enough carbohydrates to maintain blood glucose levels, enough protein to maintain body repair, and enough fat to ensure palatability. QUESTION 28 The primary reason for sending a burn client home with a pressure garment, such as a Jobst garment, is that the garment: A. Decreases hypertrophic scar formation B. Assists with ambulation C. Covers burn scars and decreases the psychological impact during recovery D. Increases venous return and cardiac output by normalizing fluid status Answer: A Explanation: (A) Tubular support, such as that received with a Jobst garment, applies tension of 10–20 mm Hg. This amount of uniform pressure is necessary to prevent or reduce hypertrophic scarring. Clients typically wear a pressure garment for 6–12 months during the recovery phase of their care. (B) Pressure garments have no ambulatory assistive properties. (C) Pressure garments can worsen the psychological impact of burn injury, especially if worn on the face. (D) Pressure garments do not normalize fluid status. QUESTION 29 The nurse would expect to include which of the following when planning the management of the client with Lyme disease? A. Complete bed rest for 6–8 weeks B. Tetracycline treatment C. IV amphotericin B D. High-protein diet with limited fluids NCLEX RN ACTUAL EXAM TEST BANK OF REAL QUESTIONS & ANSWERS 2023/2024 100%Complete Graded A+ Answer: B Explanation: (A) The client is not placed on complete bed rest for 6 weeks. (B) Tetracycline is the treatment of choice for children with Lyme disease who are over the age of 9. (C) IV amphotericin B is the treatment for histoplasmosis. (D) The client is not restricted to a high-protein diet with limited fluids. QUESTION 30 The physician recommends immediate hospital admission for a client with PIH. She says to the nurse, ―It‘s not so easy for me to just go right to the hospital like that.‖ After acknowledging her feelings, which of these approaches by the nurse would probably be best? A. Stress to the client that her husband would want her to do what is best for her health. B. Explore with the client her perceptions of why she is unable to go to the hospital. C. Repeat the physician‘s reasons for advising immediate hospitalization. D. Explain to the client that she is ultimately responsible for her own welfare and that of her baby. Answer: B Explanation: (A) This answer does not hold the client accountable for her own health. (B) The nurse should explore potential reasons for the client‘s anxiety: are there small children at home, is the husband out of town? The nurse should aid the client in seeking support or interventions to decrease the anxiety of hospitalization. (C) Repeating the physician‘s reason for recommending hospitalization may not aid the client in dealing with her reasons for anxiety. (D) The concern for self and welfare of baby may be secondary to a woman who is in a crisis situation. The nurse should explore the client‘s potential reasons for anxiety. For example, is there another child in the home who is ill, or is there a husband who is overseas and not able to return on short notice? QUESTION 31 The child with iron poisoning is given IV deforoxamine mesylate (Desferal). Following administration, the child suffers hypotension, facial flushing, and urticari A. The initial nursing intervention would be to: A. Discontinue the IV B. Stop the medication, and begin a normal saline infusion C. Take all vital signs, and report to the physician D. Assess urinary output, and if it is 30 mL an hour, maintain current treatment Answer: B Explanation: (A) The IV line should not be discontinued because other IV medications will be needed. (B) Stop the medication and begin a normal saline infusion. The child is exhibiting signs of an allergic reaction and could go into shock if the medication is not stopped. The line should be kept opened for other medication. (C) Taking vital signs and reporting to the physician is not an adequate intervention because the IV medication continues to flow. (D) Assessing urinary output and, if it is 30 mL an hour, maintaining current treatment is an inappropriate intervention owing to the child‘s obvious allergic reaction. NCLEX RN ACTUAL EXAM TEST BANK OF REAL QUESTIONS & ANSWERS 2023/2024 100%Complete Graded A+ QUESTION 32 Provide the 1-minute Apgar score for an infant born with the following findings: Heart rate: Above 100 Respiratory effort: Slow, irregular Muscle tone: Some flexion of extremities Reflex irritability: Vigorous cry Color: Body pink, blue extremities A. 7 B. 10 C. 8 D. 9 Answer: A Explanation: (A) Seven out of a possible perfect score of 10 is correct. Two points are given for heart rate above 100; 1 point is given for slow, irregular respiratory effort; 1 point is given for some flex- ion of extremities in assessing muscle tone; 2 points are given for vigorous cry in assessing reflex irritability; 1 point is assessed for color when the body is pink with blue extremities (acrocyanosis). (B) For a perfect Apgar score of 10, the infant would have a heart rate over 100 but would also have a good cry, active motion, and be completely pink. (C) For an Apgar score of 8 the respiratory rate, muscle tone, or color would need to fall into the 2-point rather than the 1-point category. (D) For this infant to receive an Apgar score of 9, four of the areas evaluated would need ratings of 2 points and one area, a rating of 1 point. QUESTION 33 A client has been diagnosed as being preeclamptic. The physician orders magnesium sulfate. Magnesium sulfate (MgSO4) is used in the management of preeclampsia for: A. Prevention of seizures B. Prevention of uterine contractions C. Sedation D. Fetal lung protection Answer: A Explanation: (A) MgSO4 is classified as an anticonvulsant drug. In preeclampsia management, MgSO4 is used for prevention of seizures. (B) MgSO4 has been used to inhibit hyperactive labor, but results are questionable. (C) Negative side effects such as respiratory depression should not be confused with generalized sedation. (D) MgSO4 does not affect lung maturity. The infant should be assessed for neuromuscular and respiratory depression. QUESTION 34 In the client with a diagnosis of coronary artery disease, the nurse would anticipate the complication of bradycardia with occlusion of which coronary artery? A. Right coronary artery B. Left main coronary artery C. Circumflex coronary artery D. Left anterior descending coronary artery NCLEX NCLEX-RN QUESTION 43 The predominant purpose of the first Apgar scoring of a newborn is to: A. ―It concerns me that you feel so badly when you have so many positive things in your life.‖ B. ―It will take a few weeks for you to feel better, so you need to be patient.‖ C. ―You are telling me that you are feeling hopeless at this point?‖ D. ―Let‘s play cards with some of the other clients to get your mind off your problems for now.‖ Answer: C Explanation: (A) This response does not acknowledge the client‘s feelings and may increase his feelings of guilt. (B) This response denotes false reassurance. (C) This response acknowledges the client‘s feelings and invites a response. (D) This response changes the subject and does not allow the client to talk about his feelings. QUESTION 41 The usual treatment for diabetes insipidus is with IM or SC injection of vasopressin tannate in oil. Nursing care related to the client receiving IM vasopressin tannate would include: A. Weigh once a week and report to the physician any weight gain of10 lb. B. Limit fluid intake to 500 mL/day. C. Store the medication in a refrigerator and allow to stand at room temperature for 30 minutes prior to administration. D. Hold the vial under warm water for 10–15 minutes and shake vigorously before drawing medication into the syringe. Answer: D Explanation: (A) Weight should be obtained daily. (B) Fluid is not restricted but is given according to urine output. (C) The medication does not have to be stored in a refrigerator. (D) Holding the vial under warm water for 10–15 minutes or rolling between your hands and shaking vigorously before drawing medication into the syringe activates the medication in the oil solution. QUESTION 42 Proper positioning for the child who is in Bryant‘s traction is: A. Both hips flexed at a 90-degree angle with the knees extended and the buttocks elevated off the bed B. Both legs extended, and the hips are not flexed C. The affected leg extended with slight hip flexion D. Both hips and knees maintained at a 90-degree flexion angle, and the back flat on the bed Answer: A Explanation: (A) The child‘s weight supplies the countertraction for Bryant‘s traction; the buttocks are slightly elevated off the bed, and the hips are flexed at a 90-degree angle. Both legs are suspended by skin traction. (B) The child in Buck‘s extension traction maintains the legs extended and parallel to the bed. (C) The child in Russell traction maintains hip flexion of the affected leg at the prescribed angle with the leg extended. (D) The child in ―90–90‖ NCLEX NCLEX-RN QUESTION 43 The predominant purpose of the first Apgar scoring of a newborn is to: traction maintains both hips and knees at a 90-degree flexion angle and the back is flat on the bed. NCLEX NCLEX-RN QUESTION 46 A client returns for her 6-month prenatal checkup and has gained 10 lb in 2 months. The results of her physical A. Determine gross abnormal motor function B. Obtain a baseline for comparison with the infant‘s future adaptation to the environment C. Evaluate the infant‘s vital functions D. Determine the extent of congenital malformations Answer: C Explanation: (A) Apgar scores are not related to the infant‘s care, but to the infant‘s physical condition. (B) Apgar scores assess the current physical condition of the infant and are not related to future environmental adaptation. (C) The purpose of the Apgar system is to evaluate the physical condition of the newborn at birth and to determine if there is an immediate need for resuscitation. (D) Congenital malformations are not one of the areas assessed with Apgar scores. QUESTION 44 Hematotympanum and otorrhea are associated with which of the following head injuries? A. Basilar skull fracture B. Subdural hematoma C. Epidural hematoma D. Frontal lobe fracture Answer: A Explanation: (A) Basilar skull fractures are fractures of the base of the skull. Blood behind the eardrum or blood or cerebrospinal fluid (CSF) leaking from the ear are indicative of a dural laceration. Basilar skull fractures are the only type with these symptoms. (B, C, D) These do not typically cause dural lacerations and CSF leakage. QUESTION 45 Which of the following risk factors associated with breast cancer would a nurse consider most significant in a client‘s history? A. Menarche after age 13 B. Nulliparity C. Maternal family history of breast cancer D. Early menopause Answer: C Explanation: (A) Women who begin menarche late (after 13 years old) have a lower risk of developing breast cancer than women who have begun earlier. Average age for menarche is 12.5 years. (B) Women who have never been pregnant have an increased risk for breast cancer, but a positive family history poses an even greater risk. (C) A positive family history puts a woman at an increased risk of developing breast cancer. It is recommended that mammography screening begin 5 years before the age at which an immediate female relative was diagnosed NCLEX NCLEX-RN decreased peripheral perfusion and bradycardi A. (B) Dobutamine‘s side effects include increased heart rate and blood pressure, ventricular ectopy, nausea, Explanation: (A) Norepinephrine‘s side effects are primarily related to safe, effective care environment and include NCLEX NCLEX-RN and headache. (C) Propranolol‘s side effects include elevated blood urea nitrogen, serum transaminase, alkaline phosphatase, and lactic dehydrogenase. (D) Epinephrine increases serum glucose levels by increasing glycogenolysis and inhibiting insulin release. Prolonged use can elevate serum lactate levels, leading to metabolic acidosis, increased urinary catecholamines, false elevation of blood urea nitrogen, and decreased coagulation time. QUESTION 49 When a client questions the nurse as to the purpose of exercise electrocardiography (ECG) in the diagnosis of cardiovascular disorders, the nurse‘s response should be based on the fact that: A. The test provides a baseline for further tests B. The procedure simulates usual daily activity and myocardial performance C. The client can be monitored while cardiac conditioning and heart toning are done D. Ischemia can be diagnosed because exercise increases O2 consumption and demand Answer: D Explanation: (A) The purpose of the study is not to provide a baseline for further tests. (B) The test causes an increase in O2 demand beyond that required to perform usual daily activities. (C) Monitoring does occur, but the test is not for the purpose of cardiac toning and conditioning. (D) Exercise ECG, or stress testing, is designed to elevate the peripheral and myocardial needs for O2 to evaluate the ability of the myocardium and coronary arteries to meet the additional demands. QUESTION 50 The following medications were noted on review of the client‘s home medication profile. Which of the medications would most likely potentiate or elevate serum digoxin levels? A. KCl B. Thyroid agents C. Quinidine D. Theophylline Answer: C Explanation: (A) Hypokalemia can cause digoxin toxicity. Administration of KCl would prevent this. (B) Thyroid agents decrease digoxin levels. (C) Quinidine increases digoxin levels dramatically. (D) Theophylline is not noted to have an effect on digoxin levels. QUESTION 51 Which of the following statements relevant to a suicidal client is correct? A. The more specific a client‘s plan, the more likely he or she is to attempt suicide. B. A client who is unsuccessful at a first suicide attempt is not likely to make future attempts. C. A client who threatens suicide is just seeking attention and is not likely to attempt suicide. D. Nurses who care for a client who has attempted suicide should not make any reference to the word ―suicide‖ in order to protect the client‘s ego. NCLEX NCLEX-RN D. Generalized swelling, pain, and diminished functional use with muscle rigidity and crepitus Answer: C Answer: A Explanation: (A) This is a high-risk factor for potential suicide. (B) A previous suicide attempt is a definite risk factor for subsequent attempts. (C) Every threat of suicide should be taken seriously. (D) The client should be asked directly about his or her intent to do bodily harm. The client is never hurt by direct, respectful questions. QUESTION 52 When assessing a child with diabetes insipidus, the nurse should be aware of the cardinal signs of: A. Anemia and vomiting B. Polyuria and polydipsia C. Irritability relieved by feeding formula D. Hypothermia and azotemia Answer: B Explanation: (A) Anemia and vomiting are not cardinal signs of diabetes insipidus. (B) Polyuria and polydipsia are the cardinal signs of diabetes insipidus. (C) Irritability relieved by feeding water, not formula, is a common sign, but not the cardinal sign, of diabetes insipidus. (D) Hypothermia and azotemia are signs, but not cardinal signs, of diabetes insipidus. QUESTION 53 The physician orders fluoxetine (Prozac) for a depressed client. Which of the following should the nurse remember about fluoxetine? A. Because fluoxetine is a tricyclic antidepressant, it may precipitate a hypertensive crisis. B. The therapeutic effect of the drug occurs 2–4 weeks after treatment is begun. C. Foods such as aged cheese, yogurt, soy sauce, and bananas should not be eaten with this drug. D. Fluoxetine may be administered safely in combination with monoamine oxidase (MAO) inhibitors. Answer: B Explanation: (A) Fluoxetine is not a tricyclic antidepressant. It is an atypical antidepressant. (B) This statement is true. (C) These foods are high in tyramine and should be avoided when the client is taking MAO inhibitors. Fluoxetine is not an MAO inhibitor. (D) Fatal reactions have been reported in clients receiving fluoxetine in combination with MAO inhibitors. QUESTION 54 A child sustains a supracondylar fracture of the femur. When assessing for vascular injury, the nurse should be alert for the signs of ischemia, which include: NCLEX NCLEX-RN D. Decreased tidal volume and tachypnea Answer: C B. Hypoxemia and respiratory acidosis C. Mediastinal tissue and organ shifting NCLEX NCLEX-RN Explanation: (A, B, D) These occur in both tension pneumothorax and open pneumothorax. (C) The tension pneumothorax acts like a one- way valve so that the pneumothorax increases with each breath. Eventually, it occupies enough space to shift mediastinal tissue toward the unaffected side away from the midline. Tracheal deviation, movement of point of maximum impulse, and decreased cardiac output will occur. The other three options will occur in both types of pneumothorax. QUESTION 58 A 38-year-old pregnant woman visits her nurse practitioner for her regular prenatal checkup. She is 30 weeks‘ gestation. The nurse should be alert to which condition related to her age? A. Iron-deficiency anemia B. Sexually transmitted disease (STD) C. Intrauterine growth retardation D. Pregnancy-induced hypertension (PIH) Answer: D Explanation: (A) Iron-deficiency anemia can occur throughout pregnancy and is not age related. (B) STDs can occur prior to or during pregnancy and are not age related. (C) Intrauterine growth retardation is an abnormal process where fetal development and maturation are delayed. It is not age related. (D) Physical risks for the pregnant client older than 35 include increased risk for PIH, cesarean delivery, fetal and neonatal mortality, and trisomy. QUESTION 59 A type I diabetic client is diagnosed with cellulitis in his right lower extremity. The nurse would expect which of the following to be present in relation to his blood sugar level? A. A normal blood sugar level B. A decreased blood sugar level C. An increased blood sugar level D. Fluctuating levels with a predawn increase Answer: C Explanation: (A) Blood sugar levels increase when the body responds to stress and illness. (B) Blood sugar levels increase when the body responds to stress and illness. (C) Hyperglycemia occurs because glucose is produced as the body responds to the stress and illness of cellulitis. (D) Blood sugar levels remain elevated as long as the body responds to stress and illness. QUESTION 60 A laboratory technique specific for diagnosing Lyme disease is: A. Polymerase chain reaction B. Heterophil antibody test C. Decreased serum calcium level D. Increased serum potassium level Answer: A NCLEX NCLEX-RN B. Exacerbation of depressive symptoms Explanation: (A) Polymerase chain reaction is the laboratory technique specific for Lyme disease. (B) Heterophil antibody test is used to diagnose mononucleosis. (C) Lyme disease does not decrease the serum calcium level. (D) Lyme disease does not increase the serum potassium level. QUESTION 61 The cardiac client who exhibits the symptoms of disorientation, lethargy, and seizures may be exhibiting a toxic reaction to: A. Digoxin (Lanoxin) B. Lidocaine (Xylocaine) C. Quinidine gluconate or sulfate (Quinaglute, Quinidex) D. Nitroglycerin IV (Tridil) Answer: B Explanation: (A) Side effects of digoxin include headache, hypotension, AV block, blurred vision, and yellow-green halos. (B) Side effects of lidocaine include heart block, headache, dizziness, confusion, tremor, lethargy, and convulsions. (C) Side effects of quinidine include heart block, hepatotoxicity, thrombocytopenia, and respiratory depression. (D) Side effects of nitroglycerin include postural hypotension, headache, dizziness, and flushing. QUESTION 62 A 27-year-old man was diagnosed with type I diabetes 3 months ago. Two weeks ago he complained of pain, redness, and tenderness in his right lower leg. He is admitted to the hospital with a slight elevation of temperature and vague complaints of ―not feeling well.‖ At 4:30 PM on the day of his admission, his blood glucose level is 50 mg; dinner will be served at 5:00 PM. The best nursing action would be to: A. Give him 3 tbsp of sugar dissolved in 4 oz of grape juice to drink B. Ask him to dissolve three pieces of hard candy in his mouth C. Have him drink 4 oz of orange juice D. Monitor him closely until dinner arrives Answer: C Explanation: (A) The combination of sugar and juice will increase the blood sugar beyond the normal range. (B) Concentrated sweets are not absorbed as fast as juice; consequently, they elevate the blood sugar beyond the normal limit. (C) Four ounces of orange juice will act immediately to raise the blood sugar to a normal level and sustain it for 30 minutes until supper is served. (D) There is an increased potential for the client‘s blood sugar to decrease even further, resulting in diabetic coma. QUESTION 63 After 3 weeks of treatment, a severely depressed client suddenly begins to feel better and starts interacting NCLEX NCLEX-RN B. Epinephrine A. Phentolamine (Regitine) NCLEX NCLEX-RN C. Phenylephrine (Neo-Synephrine) D. Sodium bicarbonate Answer: A Explanation: (A) Phentolamine is given to counteract the-adrenergic effects that cause ischemia and necrosis of local tissue. (B) Epinephrine is an endogenous catecholamine that produces vasoconstriction and increases heart rate and contractility. (C) Phenylephrine causes constriction of arterioles of skin, mucous membranes, and viscera, which in turn can cause ischemia and necrosis. (D) Sodium bicarbonate is an alkalinizing agent that is incompatible with dopamine. QUESTION 67 A male client receives 10 U of regular human insulin SC at 9:00 AM. The nurse would expect peak action from this injection to occur at: A. 9:30 AM B. 10:30 AM C. 12 noon D. 4:00 PM Answer: C Explanation: (A) This is too early for peak action to occur. (B) This is too early for peak action to occur. (C) Regular insulin peak action occurs 2–4 hours after administration. (D) This is too late for peak action to occur. QUESTION 68 As the nurse assesses a male adolescent with chlamydia, the nurse determines that a sign of chlamydia is: A. Enlarged penis B. Secondary lymphadenitis C. Epididymitis D. Hepatomegaly Answer: C Explanation: (A) An enlarged penis is not a sign of chlamydi A. (B) Secondary lymphadenitis is a complication of lymphogranuloma venereum. (C) Untreated chlamydial infection can spread from the urethra, causing epididymitis, which presents as a tender, scrotal swelling. (D) Hepatomegaly is not a complication. QUESTION 69 One of the most dramatic and serious complications associated with bacterial meningitis is Waterhouse- Friderichsen syndrome, which is: A. Peripheral circulatory collapse B. Syndrome of inappropriate antiduretic hormone C. Cerebral edema resulting in hydrocephalus NCLEX NCLEX-RN D. Auditory nerve damage resulting in permanent hearing loss Answer: A Explanation: (A) Waterhouse-Friderichsen syndrome is peripheral circulatory collapse, which may result in extensive and diffuse intravascular coagulation and thrombocytopenia resulting in death. (B) Syndrome of inappropriate antidiuretic hormone is a complication of meningitis, but it is not Waterhouse- Friderichsen syndrome. (C) Cerebral edema resulting in hydrocephalus is a complication of meningitis, but it is not Waterhouse-Friderichsen syndrome. (D) Auditory nerve damage resulting in permanent hearing loss is a complication of meningitis, but it is not Waterhouse- Friderichsen syndrome. QUESTION 70 Priapism may be a sign of: A. Altered neurological function B. Imminent death C. Urinary incontinence D. Reproductive dysfunction Answer: A Explanation: (A) Priapism in the trauma client is due to the neurological dysfunction seen in spinal cord injury. Priapism is an abnormal erection of the penis; it may be accompanied by pain and tenderness. This may disappear as spinal cord edema is relieved. (B) Priapism is not associated with death. (C) Urinary retention, rather than incontinence, may occur. (D) Reproductive dysfunction may be a secondary problem. QUESTION 71 When assessing fetal heart rate status during labor, the monitor displays late decelerations with tachycardia and decreasing variability. What action should the nurse take? A. Continue monitoring because this is a normal occurrence. B. Turn client on right side. C. Decrease IV fluids. D. Report to physician or midwife. Answer: D Explanation: (A) This is not a normal occurrence. Late decelerations need prompt intervention for immediate infant recovery. (B) To increase O2 perfusion to the unborn infant, the mother should be placed on her left side. (C) IV fluids should be increased, not decreased. (D) Immediate action is warranted, such as reporting findings, turning mother on left side, administering O2, discontinuing oxytocin (Pitocin), assessing maternal blood pressure and the labor process, preparing for immediate cesarean delivery, and explaining plan of action to client. QUESTION 72 The most commonly known vectors of Lyme disease are: A. Mites B. Fleas NCLEX NCLEX-RN C. High carbohydrate and high calorie D. Low carbohydrate and high calorie Answer: C Explanation: (A) Protein increases the workload of the liver. Increased carbohydrates provide needed calories and promote palatability. (B) Dietary intake should be adequate to ensure wound healing. (C) Increased carbohydrates provide needed calories. (D) A highcalorie diet is best obtained from carbohydrates because of their palatability. Fats increase the workload of the liver. QUESTION 79 A long-term goal for the nurse in planning care for a depressed, suicidal client would be to: A. Provide him with a safe and structured environment. B. Assist him to develop more effective coping mechanisms. C. Have him sign a ―no-suicide‖ contract. D. Isolate him from stressful situations that may precipitate a depressive episode. Answer: B Explanation: (A) This statement represents a short-term goal. (B) Long-term therapy should be directed toward assisting the client to cope effectively with stress. (C) Suicide contracts represent short-term interventions. (D) This statement represents an unrealistic goal. Stressful situations cannot be avoided in reality. QUESTION 80 The primary reason that an increase in heart rate (100 bpm) detrimental to the client with a myocardial infarction (MI) is that: A. Stroke volume and blood pressure will drop proportionately B. Systolic ejection time will decrease, thereby decreasing cardiac output C. Decreased contractile strength will occur due to decreased filling time D. Decreased coronary artery perfusion due to decreased diastolic filling time will occur, which will increase ischemic damage to the myocardium Answer: D Explanation: (A) Decreased stroke volume and blood pressure will occur secondary to decreased diastolic filling. (B) Tachycardia primarily decreases diastole; systolic time changes very little. (C) Contractility decreases owing to the decreased filling time and decreased time for fiber lengthening. (D) Decreased O2 supply due to decreased time for filling of the coronary arteriesincreases ischemia and infarct size. Tachycardia primarily robs the heart of diastolic time, which is the primary time for coronary artery filling. QUESTION 81 Which of the following nursing orders should be included in the plan of care for a client with hepatitis C? A. The nurse should use universal precautions when obtaining blood samples. B. Total bed rest should be maintained until the client is asymptomatic. C. The client should be instructed to maintain a low semi-Fowler position when eating meals. NCLEX NCLEX-RN D. The nurse should administer an alcohol backrub at bedtime. Answer: A Explanation: (A) The source of infection with hepatitis C is contaminated blood products. (B) Modified bed rest should be maintained while the client is symptomatic. Routine activities can be slowly resumed once the client is asymptomatic. (C) Nausea and vomiting occur frequently with hepatitis C. A high Fowler position may decrease the tendency to vomit. (D) The buildup of bilirubin in the client‘s skin may cause pruritus. Alcohol is a drying agent. QUESTION 82 When inspecting a cardiovascular client, the nurse notes that he needs to sit upright to breathe. This behavior is most indicative of: A. Pericarditis B. Anxiety C. Congestive heart failure D. Angina Answer: C Explanation: (A) Pericarditis can cause dyspnea but primarily causes chest pain. (B) Anxiety can cause dyspnea resulting in SOB, yet it is not typically influenced by degree of head elevation. (C) The inability to oxygenate well without being upright is most indicative of congestive heart failure, due to alveolar drowning. (D) Angina causes primarily chest pain; any SOB associated with angina is not influenced by body position. QUESTION 83 The physician has ordered that a daily exercise program be instituted by a client with type I diabetes following his discharge from the hospital. Discharge instructions about exercise should include which of the following? A. Exercise should be performed 30 minutes before meals. B. A snack may be needed before and/or during exercise. C. Hyperglycemia may occur 2–4 hours after exercise. D. The blood glucose level should be 100 mg or below before exercise is begun. Answer: B Explanation: (A) Exercise should not be performed before meals because the blood sugar is usually lower just prior to eating; therefore, there is an increased risk for hypoglycemi A. (B) Exercise lowers blood sugar levels; therefore, a snack may be needed to maintain the appropriate glucose level. (C) Exercise lowers blood sugar levels. (D) Exercise lowers blood sugar levels. If the blood glucose level is 100 mg or below at the start of exercise, the potential for hypoglycemia is greater. QUESTION 84 The nurse notes hyperventilation in a client with a thermal injury. She recognizes that this may be a reaction to which of the following medications if applied in large amounts? A. Neosporin sulfate NCLEX NCLEX-RN B. Mafenide acetate C. Silver sulfadiazine D. Povidone-iodine Answer: B Explanation: (A) The side effects of neomycin sulfate include rash, urticaria, nephrotoxicity, and ototoxicity. (B) The side effects of mafenide acetate include bone marrow suppression, hemolytic anemia, eosinophilia, and metabolic acidosis. The hyperventilation is a compensatory response to the metabolic acidosis. (C) The side effects of silver sulfadiazine include rash, itching, leukopenia, and decreased renal function. (D) The primary side effect of povidone- iodine is decreased renal function. QUESTION 85 The nurse practitioner determines that a client is approximately 9 weeks‘ gestation. During the visit, the practitioner informs the client about symptoms of physical changes that she will experience during her first trimester, such as: A. Nausea and vomiting B. Quickening C. A 6–8 lb weight gain D. Abdominal enlargement Answer: A Explanation: (A) Nausea and vomiting are experienced by almost half of all pregnant women during the first 3 months of pregnancy as a result of elevated human chorionic gonadotropin levels and changed carbohydrate metabolism. (B) Quickening is the mother‘s perception of fetal movement and generally does not occur until 18–20 weeks after the last menstrual period in primigravidas, but it may occur as early as 16 weeks in multigravidas. (C) During the first trimester there should be only a modest weight gain of 2–4 lb. It is not uncommon for women to lose weight during the first trimester owing to nausea and/or vomiting. (D) Physical changes are not apparent until the second trimester, when the uterus rises out of the pelvis. QUESTION 86 Which of the following medications requires close observation for bronchospasm in the client with chronic obstructive pulmonary disease or asthma? A. Verapamil (Isoptin) B. Amrinone (Inocor) C. Epinephrine (Adrenalin) D. Propranolol (Inderal) Answer: D Explanation: (A) Verapamil has the respiratory side effect of nasal or chest congestion, dyspnea, shortness of breath (SOB), and wheezing. (B) Amrinone has the effect of increased contractility and dilation of the vascular smooth muscle. It has no noted respiratory side effects. (C) Epinephrine has the effect of bronchodilation through stimulation. (D) Propranolol, esmolol, and labetalol are all - blocking agents, which can increase airway resistance and cause bronchospasms. NCLEX NCLEX-RN Answer: C Explanation: (A)March 27 is a miscalculation. (B) February 1 is a miscalculation. (C) February 27 is the correct answer. To calculate the estimated date of confinement using Nagele‘s rule, subtract 3 months from the date that the last menstrual cycle began and then add 7 days to the result. (D) January 3 is a miscalculation. QUESTION 93 The therapeutic blood-level range for lithium is: A. 0.25–1.0 mEq/L B. 0.5–1.5 mEq/L C. 1.0–2.0 mEq/L D. 2.0–2.5 mEq/L Answer: B Explanation: (A) This range is too low to be therapeutic. (B) This is the therapeutic range for lithium. (C) This range is above the therapeutic level. (D) This range is toxic and may cause severe side effects. QUESTION 94 Which of the following ECG changes would be seen as a positive myocardial stress test response? A. Hyperacute T wave B. Prolongation of the PR interval C. ST-segment depression D. Pathological Q wave Answer: C Explanation: (A) Hyperacute T waves occur with hyperkalemi A. (B) Prolongation of the P R interval occurs with first-degree AV block. (C) Horizontal ST-segment depression of>1 mm during exercise isdefinitely a positive criterion on the exercise ECG test. (D) Patho-logical Q waves occur with MI. QUESTION 95 A psychotic client who believes that he is God and rules all the universe is experiencing which type of delusion? A. Somatic B. Grandiose C. Persecutory D. Nihilistic Answer: B Explanation: (A) These delusions are related to the belief that an individual has an incurable illness. (B) These delusions are related to feelings of self-importance and uniqueness. (C) These delusions are related to feelings of being conspired against. (D) These delusions are related to denial of self-existence. NCLEX NCLEX-RN QUESTION 96 During burn therapy, morphine is primarily administered IV for pain management because this route: A. Delays absorption to provide continuous pain relief B. Facilitates absorption because absorption from muscles is not dependable C. Allows for discontinuance of the medication if respiratory depression develops D. Avoids causing additional pain from IM injections Answer: B Explanation: (A) Absorption would be increased, not decreased. (B) IM injections should not be used until the client is hemodynamically stable and has adequate tissue perfusion. Medications will remain in the subcutaneous tissue with the fluid that is present in the interstitial spaces in the acute phase of the thermal injury. The client will have a poor response to the medication administered, and a ―dumping‖ of the medication can occur when the medication and fluid are shifted back into the intravascular spaces in the next phase of healing. (C) IV administration of the medication would hasten respiratory compromise, if present. (D) The desire to avoid causing the client additional pain is not a primary reason for this route of administration. QUESTION 97 Assessment of the client with pericarditis may reveal which of the following? A. Ventricular gallop and substernal chest pain B. Narrowed pulse pressure and shortness of breath C. Pericardial friction rub and pain on deep inspiration D. Pericardial tamponade and widened pulse pressure Answer: C Explanation: (A) No S3 or S4 are noted with pericarditis. (B) No change in pulse pressure occurs. (C) The symptoms of pericarditis vary with the cause, but they usually include chest pain, dyspnea, tachycardia, rise in temperature, and friction rub caused by fibrin or other deposits. The pain seen with pericarditis typically worsens with deep inspiration. (D) Tamponade is not typically seen early on, and no change in pulse pressure occurs. QUESTION 98 A client who has sustained a basilar skull fracture exhibits blood-tinged drainage from his nose. After establishing a clear airway, administering supplemental O2, and establishing IV access, the next nursing intervention would be to: A. Pass a nasogastric tube through the left nostril B. Place a 4 X 4 gauze in the nares to impede the flow C. Gently suction the nasal drainage to protect the airway D. Perform a halo test and glucose level on the drainage Answer: D Explanation: (A) Basilar skull fracture may cause dural lacerations, which result in CSF leaking from the ears or nose. Insertion of a tube could lead to CSF going into the brain tissue or sinuses. (B) Tamponading flow could worsen the problem and increase ICP. (C) Suction could increase brain damage NCLEX NCLEX-RN and dislocate tissue. (D) Testing the fluid from the nares would determine the presence of CSF. Elevation of the head, notification of the medical staff, and prophylactic antibiotics are appropriate therapy. QUESTION 99 To appropriately monitor therapy and client progress, the nurse should be aware that increased myocardial work and O2 demand will occur with which of the following? A. Positive inotropic therapy B. Negative chronotropic therapy C. Increase in balance of myocardial O2 supply and demand D. Afterload reduction therapy Answer: A Explanation: (A) Inotropic therapy will increase contractility, which will increase myocardial O2 demand. (B) Decreased heart rate to the point of bradycardia will increase coronary artery filling time. This should be used cautiously because tachycardia may be a compensatory mechanism to increase cardiac output. (C) The goal in the care of the MI client with angina is to maintain a balance between myocardial O2 supply and demand. (D) Decrease in systemic vascular resistance by drug therapy, such as IV nitroglycerin or nitroprusside, or intra-aortic balloon pump therapy, would decrease myocardial work and O2 demand. QUESTION 100 The most important reason to closely assess circumferential burns at least every hour is that they may result in: A. Hypovolemia B. Renal damage C. Ventricular arrhythmias D. Loss of peripheral pulses Answer: D Explanation: (A) Hypovolemia could be a result of fluid loss from thermal injury, but not as a result of the circumferential injury. (B) Renal damage is typically seen because of prolonged hypovolemia or myoglobinuri A. (C) Electrical injuries and electrolyte changes typically cause arrhythmias in the burn client. (D) Full- thickness circumferential burns are nonelastic and result in an internal tourniquet effect that compromises distal blood flow when the area involved is an extremity.Circumferential full-thickness torso burns compromise respiratory motion and, when extreme, cardiac return. QUESTION 101 A pregnant client comes to the office for her first prenatal examination at 10 weeks. She has been pregnant twice before; the first delivery produced a viable baby girl at 39 weeks 3 years ago; the second pregnancy produced a viable baby boy at 36 weeks 2 years ago. Both children are living and well. Using the GTPAL system to record her obstetrical history, the nurse should record: A. 3-2-0-0-2 B. 2-2-0-2-2 C. 3-1-1-0-2 NCLEX NCLEX-RN QUESTION 107 The nurse working in a prenatal clinic needs to be alert to the cardinal signs and symptoms of PIH because: A. Immediate treatment of mild PIH includes the administration of a variety of medications B. Psychological counseling is indicated to reduce the emotional stress causing the blood pressure elevation C. Self-discipline is required to control caloric intake throughout the pregnancy D. The client may not recognize the early symptoms of PIH Answer: D Explanation: (A) Mild PIH is not treated with medications. (B) Emotional stress is not the cause of blood pressure elevation in PIH. (C) Excessive caloric intake is not the cause of weight gain in PIH. (D) The client most frequently is not aware of the signs and symptoms in mild PIH. QUESTION 108 A client is admitted to the labor unit. On vaginal examination, the presenting part in a cephalic presentation was at station plus two. Station 12 means that the: A. Presenting part is 2 cm above the level of the ischial spines B. Biparietal diameter is at the level of the ischial spines C. Presenting part is 2 cm below the level of the ischial spines D. Biparietal diameter is 5 cm above the ischial spines Answer: C Explanation: (A) Station is the relationship of the presenting part to an imaginary line drawn between the ischial spines. If the presenting part is above the ischial spines, the station is negative. (B) When the biparietal diameter is at the level of the ischial spines, the presenting part is generally at a +4 or +5 station. (C) Station is the relationship of the presenting part to an imaginary line drawn between the ischial spines. If the presenting part is below the ischial spines, the station is positive. Thus, 2 cm below the ischial spines is the station +2. (D) When the biparietal diameter is above the ischial spines by 5 cm, the presenting part is usually engaged or at station 0. QUESTION 109 To prevent fungal infections of the mouth and throat, the nurse should teach clients on inhaled steroids to: A. Rinse the plastic holder that aerosolizes the drug with hydrogen peroxide every other day B. Rinse the mouth and gargle with warm water after each use of the inhaler C. Take antacids immediately before inhalation to neutralize mucous membranes and prevent infection D. Rinse the mouth before each use to eliminate colonization of bacteria Answer: B Explanation: (A) It is sufficient to rinse the plastic holders with warm water at least once per day. (B) It is important to rinse the mouth after each use to minimize the risk of fungal infections by reducing the droplets of the glucocorticoid left in the oral cavity. (C) Antacids act by neutralizing or reducing gastric acid, thus decreasing the pH of the stomach. ―Neutralizing‖ the oral mucosa prior to inhalation of a steroid inhaler does not minimize the risk of fungal NCLEX NCLEX-RN infections. (D) Rinsing prior to the use of the glucocorticoid will not eliminate the droplets left on the oral mucous membranes following the use of the inhaler. QUESTION 110 A 16-year-old client comes to the prenatal clinic for her monthly appointment. She has gained 14 lb from her 7th to 8th month; her face and hands indicate edem A. She is diagnosed as having PIH and referred to the high-risk prenatal clinic. The client‘s weight increase is most likely due to: A. Overeating and subsequent obesity B. Obesity prior to conception C. Hypertension due to kidney lesions D. Fluid retention Answer: D Explanation: (A) Overeating can lead to obesity, but not to edem A. (B) There is no indication of obesity prior to pregnancy. PIH is more prevalent in the underweight than in the obese in this age group. (C) Hypertension can be due to kidney lesions, but it would have been apparent earlier in the pregnancy. (D) The weight gain in PIH is due to the retention of sodium ions and fluid and is one of the three cardinal symptoms of PIH. QUESTION 111 A 6-month-old infant has developmental delays. His weight falls below the 5th percentile when plotted on a growth chart. A diagnosis of failure to thrive is made. What behaviors might indicate the possibility of maternal deprivation? A. Responsive to touch, wants to be held B. Uncomforted by touch, refuses bottle C. Maintains eye-to-eye contact D. Finicky eater, easily pacified, cuddly Answer: B Explanation: (A) Normal infant attachment behaviors include responding to touch and wanting to be held. (B) Maternal deprivation behaviors include poor feeding, stiffening and refusal to eat, and inconsistencies in responsiveness. (C) Attachment behavior includes maintaining eye contact. (D) Maternal deprivation behaviors include displeasure with touch and physical contact. QUESTION 112 The nurse teaches a male client ways to reduce the risks associated with furosemide therapy. Which of the following indicates that he understands this teaching? A. ―I‘ll be sure to rise slowly and sit for a few minutes after lying down.‖ B. ―I‘ll be sure to walk at least 2–3 blocks every day.‖ C. ―I‘ll be sure to restrict my fluid intake to four or five glasses a day.‖ D. ―I‘ll be sure not to take any more aspirin while I am NCLEX NCLEX-RN on this drug.‖ Answer: A Explanation: (A) This response will help to prevent the occurrence of postural hypotension, a common side effect of this drug and a common reason for falls. (B) Although walking is an excellent exercise, it is not specific to the reduction of risks associated with diuretic therapy. (C) Clients on diuretic therapy are generally taught to ensure that their fluid intake is at least 2000–3000 mL daily, unless contraindicated. (D) Aspirin is a safe drug to take along with furosemide. QUESTION 113 A client is admitted to the hospital for an induction of labor owing to a gestation of 42 weeks confirmed by dates and ultrasound. When she is dilated 3 cm, she has a contraction of 70 seconds. She is receiving oxytocin. The nurse‘s first intervention should be to: A. Check FHT B. Notify the attending physician C. Turn off the IV oxytocin D. Prepare for the delivery because the client is probably in transition Answer: C Explanation: (A) FHT should be monitored continuously with an induction of labor; this is an accepted standard of care. (B) The physician should be notified, but this is not the first intervention the nurse should do. (C) The standard of care for an induction according to the Association of Women‘s Health, Obstetric, and Neonatal Nurses and American College of Obstetrics and Gynecology is that contractions should not exceed 60 seconds in an induction. Inductions should simulate normal labor; 70-second contractions during the latent phase (3 cm) are not the norm. The next contractions can be longer and increase risks to the mother and fetus. (D) Contractions lasting 60–90 seconds during transition are typical; this provides a good distractor. The nurse needs to be knowledgeable of the phases and stages of labor. QUESTION 114 A male client is scheduled for a liver biopsy. In preparing him for this test, the nurse should: A. Explain that he will be kept NPO for 24 hours before the exam B. Practice with him so he will be able to hold his breath for 1 minute C. Explain that he will be receiving a laxative to prevent a distended bowel from applying pressure on the liver D. Explain that his vital signs will be checked frequently after the test Answer: D Explanation: (A) There is no NPO restriction prior to a liver biopsy. (B) The client would need to hold his breath for 5–10 seconds. (C) There is no pretest laxative given. (D) Following the test, the client is watched for hemorrhage and shock. QUESTION 115 After a liver biopsy, the best position for the client is: NCLEX NCLEX-RN (D) Fluid intake needs to be in the 2000–2500 mL range if possible to help irrigate the bladder and prevent infection. QUESTION 121 A 23-year-old borderline client is admitted to an inpatient psychiatric unit following an impulsive act of self- mutilation. A few hours after admission, she requests special privileges, and when these are not granted, she stands up and angrily shouts that the people on the unit do not care, and she storms across the room. The nurse should respond to this behavior by: A. Placing her in seclusion until the behavior is under control B. Walking up to the client and touching her on the arm to get her attention C. Communicating a desire to assist the client to regain control, offering a one-to-one session in a quiet area D. Confronting the client, letting her know the consequences for getting angry and disrupting the unit Answer: C Explanation: (A) Threatening a client with punitive action is violating a client‘s rights and could escalate the client‘s anger. (B) Angry clients need respect for personal space, and physical contact may be perceived as a threatening gesture escalating anger. (C) Client lacks sufficient self-control to limit own maladaptive behavior; she may need assistance from staff. (D) Confronting an angry client may escalate her anger to further acting out, and consequences are for acting out anger aggressively, not for getting angry or feeling angry. QUESTION 122 A client is being discharged with albuterol (Proventil) and beclomethasone dipropionate (Vanceril) to be administered via inhalation three times a day and at bedtime. Client teaching regarding the sequential order in which the drugs should be administered includes: A. Glucocorticoid followed by the bronchodilator B. Bronchodilator followed by the glucocorticoid C. Alternate successive administrations D. According to the client‘s preference Answer: B Explanation: (A) The client would not receive therapeutic effects of the glucocorticoid when it is inhaled through constricted airways. (B) Bronchodilating the airways first allows for the glucocorticoid to be inhaled through open airways and increases the penetration of the steroid for maximum effectiveness of the drug. (C) Inac- Inaccurate use of the inhalers will lead to decreased effectiveness of the treatment. (D) Client teaching regarding the use and effects of inhalers will promote client understanding and compliance. QUESTION 123 A gravida 2 para 1 client delivered a full-term newborn 12 hours ago. The nurse finds her uterus to be boggy, high, and deviated to the right. The most appropriate nursing action is to: A. Notify the physician B. Place the client on a pad count C. Massage the uterus and re-evaluate in 30 minutes D. Have the client void and then re-evaluate the fundus NCLEX NCLEX-RN Answer: D Explanation: (A) The nurse should initiate actions to remove the most frequent cause of uterine displacement, which involves emptying the bladder. Notifying the physician is an inappropriate nursing action. (B) The pad count gives an estimate of blood loss, which is likely to increase with a boggy uterus; but this action does not remove the most frequent cause of uterine displacement, which is a full bladder. (C) Massage may firm the uterus temporarily, but if a full bladder is not emptied, the uterus will remain displaced and is likely to relax again. (D) The most common cause of uterine displacement is a full bladder. QUESTION 124 During a client‘s first postpartum day, the nurse assessed that the fundus was located laterally to the umbilicus. This may be due to: A. Endometritis B. Fibroid tumor on the uterus C. Displacement due to bowel distention D. Urine retention or a distended bladder Answer: D Explanation: (A, B) Endometritis, urine retention, or bladder distention provide good distractors because they may delay involution but do not usually cause the uterus to be lateral. (C) Bowel distention and constipation are common in the postpartum period but do not displace the uterus laterally. (D) Urine retention or bladder distention commonly displaces the uterus to the right and may delay involution. QUESTION 125 The nurse instructs a pregnant client (G2P1) to rest in a side-lying position and avoid lying flat on her back. The nurse explains that this is to avoid ―vena caval syndrome,‖ a condition which: A. Occurs when blood pressure increases sharply with changes in position B. Results when blood flow from the extremities is blocked or slowed C. Is seen mainly in first pregnancies D. May require medication if positioning does not help Answer: B Explanation: (A) Blood pressure changes are predominantly due to pressure of the gravid uterus. (B) Pressure of the gravid uterus on the inferior vena cava decreases blood return from lower extremities. (C) Inferior vena cava syndrome is experienced in the latter months of pregnancy regardless of parity. (D) There are no medications useful in the treatment of interior vena cava syndrome; alleviating pressure by position changes is effective. QUESTION 126 The nurse enters the playroom and finds an 8-year-old child having a grand mal seizure. Which one of the following actions should the nurse take? A. Place a tongue blade in the child‘s mouth. NCLEX NCLEX-RN B. Restrain the child so he will not injure himself. C. Go to the nurses station and call the physician. D. Move furniture out of the way and place a blanket under his head. Answer: D Explanation: (A) The nurse should not put anything in the child‘s mouth during a seizure; this action could obstruct the airway. (B) Restraining the child‘s movements could cause constrictive injury. (C) Staying with the child during a seizure provides protection and allows the nurse to observe the seizure activity. (D) The nurse should provide safety for the child by moving objects and protecting the head. QUESTION 127 A mother frantically calls the emergency room (ER) asking what to do about her 3-year-old girl who was found eating pills out of a bottle in the medicine cabinet. The ER nurse tells the mother to: A. Give the child 15 mL of syrup of ipecac. B. Give the child 10 mL of syrup of ipecac with a sip of water. C. Give the child 1 cup of water to induce vomiting. D. Bring the child to the ER immediately. Answer: D Explanation: (A) Before giving any emetic, the substance ingested must be known. (B) At least 8 oz of water should be administered along with ipecac syrup to increase volume in the stomach and facilitate vomiting. (C) Water alone will not induce vomiting. An emetic is necessary to facilitate vomiting. (D) Vomiting should never be induced in an unconscious client because of the risk of aspiration. QUESTION 128 A mother continues to breast-feed her 3-month-old infant. She tells the nurse that over the past 3 days she has not been producing enough milk to satisfy the infant. The nurse advises the mother to do which of the following? A. ―Start the child on solid food.‖ B. ―Nurse the child more frequently during this growth spurt.‖ C. ―Provide supplements for the child between breastfeeding so you will have enough milk.‖ D. ―Wait 4 hours between feedings so that your breasts will fill up.‖ Answer: B Explanation: (A) Solid foods introduced before 4–6 months of age are not compatible with the abilities of the GI tract and the nutritional needs of the infant. (B) Production of milk is supply and demand. A common growth spurt occurs at 3 months of age, and more frequent nursing will increase the milk supply to satisfy the infant. (C) Supplementation will decrease the infant‘s appetite and in turn decrease the milk supply. When the infant nurses less often or with less vigor, the amount of milk produced decreases. (D) Rigid feeding schedules lead to a decreased milk supply, whereas frequent nursing signals the mother‘s body to produce a correspondingly increased amount of milk. NCLEX NCLEX-RN Answer: C Explanation: (A) Anorexia and weight loss are problems that need attention in severe depression, but they can be addressed secondary to immediate concerns. (B) Impaired thinking and confusion are problems in severe depression that are addressed with administration of medication, through group and individual psychotherapy, and through activity therapy as motivation and interest increase. (C) Possible harm to self as with suicidal ideation; a suicide plan, means to execute plan; and/or overt gestures or an attempt must be addressed as an immediate concern and safety measures implemented appropriate to the risk of suicide. (D) Rest and activity impairment may take time and further assessment to determine client‘s sleep pattern and amount of psychomotor retardation with the more immediate concern for safety present. QUESTION 135 The nurse is teaching a mother care of her child‘s spica cast. The mother states that he complains of itching under the edge of the cast. One nonpharmacological technique the nurse might suggest would be: A. ―Blowing air under the cast using a hair dryer on cool setting often relieves itching.‖ B. ―Slide a ruler under the cast and scratch the area.‖ C. ―Guide a towel under and through the cast and move it back and forth to relieve the itch.‖ D. ―Gently thump on cast to dislodge dried skin that causes the itching.‖ Answer: A Explanation: (A) Cool air will often relieve pruritus without damaging the cast or irritating the skin. (B) The nurse should never force anything under the cast, because the cast may become damaged and skin breakdown may occur. (C) Forcing an object under the cast could lead to cast damage and skin breakdown. The object may become lodged under the cast necessitating cast removal. (D) This technique does not dislodge skin cells. It could damage the cast and cause skin breakdown. QUESTION 136 A 56-year-old psychiatric inpatient has had recurring episodes of depression and chronic low self-esteem. She feels that her family does not want her around, experiences a sense of helplessness, and has a negative view of herself. To assist the client in focusing on her strengths and positive traits, a strategy used by the nurse would be to: A. Tell the client to attend all structured activities on the unit B. Encourage or direct client to attend activities that offer simple methods to attain success C. Increase the client‘s self-esteem by asking that she make all decisions regarding attendance in group activities D. Not allow any dependent behaviors by the client because she must learn independence and will have to ask for any assistance from staff Answer: B Explanation: (A) The nurse should encourage activities gradually, as client‘s energy level and tolerance for shared activities improve. (B) Activities that focus on strengths and accomplishments, with uncomplicated tasks, minimize failure and increase self-worth. (C) Asking a client to set a goal to make all decisions about attending group activities is unrealistic, and such decisions are not always under the client‘s control; this sets up the client for NCLEX NCLEX-RN further failure and possibly decreased self-worth. (D) Encouragement toward independence does promote increased feelings of selfworth; however, clients may need assistance with decision making and problem solving for various situations and on an individual basis. QUESTION 137 Respiratory function is altered in a 16-year-old asthmatic. Which of the following is the cause of this alteration? A. Altered surfactant production B. Paradoxical movements of the chest wall C. Increased airway resistance D. Continuous changes in respiratory rate and depth Answer: C Explanation: (A) Altered surfactant production is found in sudden infant death syndrome. (B) Paradoxical breathing occurs when a negative intrathoracic pressure is transmitted to the abdomen by a weakened, poorly functioning diaphragm. (C) Asthma is characterized by spasm and constriction of the airways resulting in increased resistance to airflow. (D) If the pulmonary tree is obstructed for any reason, inspired air has difficulty overcoming the resistance and getting out. The rate of respiration increases in order to compensate, thus increasing air exchange. QUESTION 138 A 5-year-old child has suffered second-degree thermal burns over 30% of her body. Forty-eight hours after the burn injury, the nurse must begin to monitor the child for which one of the following complications? A. Fluid volume deficit B. Fluid volume excess C. Decreased cardiac output D. Severe hypotension Answer: B Explanation: (A) Fluid volume deficit resulting from fluid shifts to the interstitial spaces occurs in the first 48 hours. (B) Forty-eight hours to 72 hours after the burn injury and fluid resuscitation, capillary permeability is restored and fluid requirements decrease. Interstitial fluid returns rapidly to the vascular compartment, and the nurse must monitor the child for signs and symptoms of hypervolemi A. (C) Increased cardiac output results as fluids shift back to the vascular compartment. (D) Hypertension is the result of hypervolemia. QUESTION 139 Prenatal clients are routinely monitored for early signs of pregnancy-induced hypertension (PIH). For the prenatal client, which of the following blood pressure changes from baseline would be most significant for the nurse to report as indicative of PIH? A. 136/88 to 144/93 B. 132/78 to 124/76 C. 114/70 to 140/88 NCLEX NCLEX-RN D. 140/90 to 148/98 Answer: C Explanation: (A) These blood pressure changes reflect only an 8 mm Hg systolic and a 5 mm Hg diastolic increase, which is insufficient for blood pressure changes indicating PIH. (B) These blood pressure changes reflect a decrease in systolic pressure of 8 mm Hg and diastolic pressure of 2 mm Hg; these values are not indicative of blood pressure increases reflecting PIH. (C) The definition of PIH is an increase in systolic blood pressure of 30 mm Hg and/or diastolic blood pressure of 15 mm Hg. These blood pressures reflect a change of 26 mm Hg systolically and 18mm Hg diastolically. (D) These blood pressures reflect a change of only 8 mm Hg systolically and 8 mm Hg diastolically, which is insufficient for blood pressure changes indicating PIH. QUESTION 140 An 83-year-old client has been hospitalized following a fall in his home. He has developed a possible fecal impaction. Which of the following assessment findings would be most indicative of a fecal impaction? A. Boardlike, rigid abdomen B. Loss of the urge to defecate C. Liquid stool D. Abdominal pain Answer: C Explanation: (A) A boardlike, rigid abdomen would point to a perforated bowel, not a fecal impaction. (B) When a client is fecally impacted, a common symptom is the urge to defecate but the inability to do so. (C) When an impaction is present, only liquid stool will be able to pass around the impacted site. (D) Abdominal pain without distention is not a sign of a fecal impaction. QUESTION 141 A postpartum client complains of rectal pressure and severe pain in her perineum; this may be indicative of: A. Afterbirth pains B. Constipation C. Cystitis D. A hematoma of the vagina or vulva Answer: D Explanation: (A) Afterbirth pains are a common complaint in the postpartum client, but they are located in the uterus. (B) Constipation may cause rectal pressure but is not usually associated with ―severe pain.‖ (C) Cystitis may cause pain, but the location is different. (D) Hematomas are frequently associated with severe pain and pressure. Further assessments are indicated for this client. QUESTION 142 Iron dextran (Imferon) is a parenteral iron preparation. The nurse should know that it: NCLEX NCLEX-RN QUESTION 148 Which of the following signs might indicate a complication during the labor process with vertex presentation? A. Fetal tachycardia to 170 bpm during a contraction B. Nausea and vomiting at 8–10 cm dilation C. Contraction lasting 60 seconds D. Appearance of dark-colored amniotic fluid Answer: D Explanation: (A) Fetal tachycardia may indicate fetal hypoxia; however, 170 bpm is only mild tachycardi A. (B) Nausea and vomiting occur frequently during transition and are not a complication. (C) Contractions frequently last 60–90 seconds during the transitional phase of labor and are not considered a complication as long as the uterus relaxes completely between contractions. (D) Passage of meconium in a vertex presentation is a sign of fetal distress; this may be normal in a breech presentation owing to pressure on the presenting part. QUESTION 149 The nurse would be concerned if a client exhibited which of the following symptoms during her postpartum stay? A. Pulse rate of 50–70 bpm by her third postpartum day B. Diuresis by her second or third postpartum day C. Vaginal discharge or rubra, serosa, then rubra D. Diaphoresis by her third postpartum day Answer: C Explanation: (A) Bradycardia is an expected assessment during the postpartum period. (B) Diuresis can occur during labor and the postpartum period and is an expected physiological adaptation. (C) A return of rubra after the serosa period may indicate a postpartal complication. (D) Diaphoresis, especially at night, is an expected physiological change and does not indicate an infectious process. Bradycardia, diuresis, and diaphoresis are normal postpartum physiological responses to adjust the cardiac output and blood volume to the nonpregnant state. QUESTION 150 MgSO4 blood levels are monitored and the nurse would be prepared to administer the following antidote for MgSO4 side effects or toxicity: A. Magnesium oxide B. Calcium hydroxide C. Calcium gluconate D. Naloxone (Narcan) Answer: C Explanation: (A, B) These drugs are not antidotes for MgSO4. (C) This drug is the standard antidote and should always be readily available when MgSO4is being administered. (D) This drug is an antidote for narcotics, not MgSO4. NCLEX NCLEX-RN QUESTION 151 A mother brings her 3-year-old child who is unconscious but breathing to the ER with an apparent drug overdose. The mother found an empty bottle of aspirin next to her child in the bathroom. Which nursing action is the most appropriate? A. Put in a nasogastric tube and lavage the child‘s stomach. B. Monitor muscular status. C. Teach mother poison prevention techniques. D. Place child on respiratory assistance. Answer: A Explanation: (A) The immediate treatment for drug overdose is removal of the drug from the stomach by either forced emesis or gastric lavage. The child‘s unconscious state prohibits forced emesis. (B) Toxic amounts of salicylates directly affect the respiratory system, which could lead to respiratory failure. (C) The mother‘s anxiety is probably so high that preventive guidance will be ineffective. (D) Respiratory assistance is not needed if the child‘s respiratory function is unaltered. QUESTION 152 Azulfidine (Sulfasalazine) may be ordered for a client who has ulcerative colitis. Which of the following is a nursing implication for this drug? A. Limit fluids to 500 mL/day. B. Administer 2 hours before meals. C. Observe for skin rash and diarrhe A. D. Monitor blood pressure, pulse. Answer: C Explanation: (A) Fluids up to 2500–3000 mL/day are needed to prevent kidney stones. (B) The client should be instructed to take oral preparations with meals or snacks to lessen gastric irritation. (C) Sulfasalazine causes skin rash and diarrhe A. (D) Blood pressure and pulse are not altered by sulfasalazine. QUESTION 153 A dose of theophylline may need to be altered if a client with COPD: A. Is allergic to morphine B. Has a history of arthritis C. Operates machinery D. Is concurrently on cimetidine for ulcers Answer: D Explanation: (A) The effects of morphine or an allergic response to the drug will not affect theophylline clearance. (B) Xanthines are used cautiously in clients with severe cardiac disease, liver disease, cor pulmonale, hypertension, or hyperthyroidism. Arthritis does not influence the dosage of theophylline. (C) Theophylline does not cause NCLEX NCLEX-RN sedation or drowsiness. Conversely, its side effects may be exhibited by central nervous system stimulation. (D) Cimetidine decreases theophylline clearance from the system and increases theophylline levels in the blood, thus increasing the risk of toxicity. QUESTION 154 A 56-year-old client is admitted to the psychiatric unit in a state of total despair. She feels hopeless and worthless, has a flat affect and very sad appearance, and is unable to feel pleasure from anything. Her husband has been assisting her at home with the housework and cooking; however, she has not been eating much, lies around or sits in a chair most of the day, and is becoming confused and thinks her family does not want her around anymore. In assessing the client, the nurse determines that her behavior is consistent with: A. Transient depression B. Mild depression C. Moderate depression D. Severe depression Answer: D Explanation: (A) Transient depression manifests as sadness or the ―blues‖ as seen with everyday disappointments and is not necessarily dysfunctional. (B) Mild depression manifests as symptoms seen with grief response, such as denial, sadness, withdrawal, somatic symptoms, and frequent or continuous thoughts of the loss. (C) Moderate depression manifests as feelings of sadness, negativism; low self-esteem; rumination about life‘s failures; decreased interest in grooming and eating; and possibly sleep disturbances. These symptoms are consistent with dysthymi A. (D) Severe depression manifests as feelings of total despair, hopelessness, emptiness, inability to feel pleasure; possibly extreme psychomotor retardation; inattention to hygiene; delusional thinking; confusion; self- blame; and suicidal thoughts. These symptoms are consistent with major depression. QUESTION 155 A client is pregnant with her second child. Her last menstrual period began on January 15. Her expected date of delivery would be: A. October 8 B. October 15 C. October 22 D. October 29 Answer: C Explanation: (A) Incorrect application of Nägele‘s rule: correctly subtracted 3 months but subtracted 7 days rather than added. (B) Incorrect application of Nägele‘s rule: correctly subtracted 3 months but did not add 7 days. (C) Correct application of Nägele‘s rule: correctly subtracted 3 months and added 7 days. (D) Incorrect application of Nägele‘s rule: correctly subtracted 3 months but added 14 days instead of 7 days. QUESTION 156 Which of the following changes in blood pressure readings should be of greatest concern to the nurse when assessing a prenatal client? NCLEX NCLEX-RN A. (D) By inserting a finger into the infant‘s mouth beside the nipple, the lactating mother can break the suction and the nipple can be removed without trauma. QUESTION 162 A parent told the public health nurse that her 6-year-old son has been taking tetracycline for a chronic skin condition. The parent asked if this could cause any problems for the child. What should the nurse explain to the parent? A. Giving tetracycline to a child younger than 8 years may cause permanent staining of his teeth. B. If you give tetracycline with milk, it may be absorbed readily. C. The medication should be given to adults, not children. D. Secondary infections of chronic skin disorders do not respond to antibiotics. Answer: A Explanation: (A) Tetracycline should be avoided during tooth development because it interferes with enamel formation and dental pigmentation. (B) Milk interferes with the absorption of tetracyclines. (C) Children older than 9 years or past the tooth development stage may be given tetracycline. (D) Secondary infections of chronic skin disorders may respond to antibiotics such as penicillin or tetracyclines. QUESTION 163 MgSO4 is ordered IV following the established protocol for a client with severe PIH. The anticipated effects of this therapy are anticonvulsant and: A. Vasoconstrictive B. Vasodilative C. Hypertensive D. Antiemetic Answer: B Explanation: (A) An anticonvulsant effect is the goal of drug therapy for PIH. However, we would not want to increase the vasoconstriction that is already present. This would make the symptoms more severe. (B) An anticon-vulsant effect and vasodilation are the desired outcomes when administering this drug. (C) An anticonvulsant effect is the goal of drug therapy for PIH; however, hypertensive drugs would increase the blood pressure even more. (D) An anticonvulsant effect is the goal of drug therapy for PIH. MgSO4is not classified as an antiemetic. Antiemetics are not indicated for PIH treatment. QUESTION 164 The nurse is preparing a 6-year-old child for an IV insertion. Which one of the following statements by the nurse is appropriate when preparing a child for a potentially painful procedure? A. ―Some say this feels like a pinch or a bug bite. You tell me what it feels like.‖ B. ―This is going to hurt a lot; close your eyes and hold my hand.‖ C. ―This is a terrible procedure, so don‘t look.‖ D. ―This will hurt only a little; try to be a big boy.‖ NCLEX NCLEX-RN Answer: A Explanation: (A) Educating the child about the pain may lessen anxiety. The child should be prepared for a potentially painful procedure but avoid suggesting pain. The nurse should allow the child his own sensory perception and evaluation of the procedure. (B) The nurse should avoid absolute descriptive statements and allow the child his own perception of the procedure. (C) The nurse should avoid evaluative statements or descriptions and give the child control in describing his reactions. (D) False statements regarding a painful procedure will cause a loss of trust between the child and the nurse. QUESTION 165 A 74-year-old female client is 3 days postoperative. She has an indwelling catheter and has been progressing well. While the nurse is in the room, the client states, ―Oh dear, I feel like I have to urinate again!‖ Which of the following is the most appropriate initial nursing response? A. Assure her that this is most likely the result of bladder spasms. B. Check the collection bag and tubing to verify that the catheter is draining properly. C. Instruct her to do Kegel exercises to diminish the urge to void. D. Ask her if she has felt this way before. Answer: B Explanation: (A) Although this may be an appropriate response, the initial response would be to assure the patency of the catheter. (B) The most frequent reason for an urge to void with an indwelling catheter is blocked tubing. This response would be the best initial response. (C) Kegel exercises while a retention catheter is in place would not help to prevent a voiding urge and could irritate the urethral sphincter. (D) Though the nurse would want to ascertain whether the client has felt the same urge to void before, the initial response should be to assure the patency of the catheter. QUESTION 166 A six-month-old infant is receiving ribavirin for the treatment of respiratory syncytial virus. Ribavirin is administered via which one of the following routes? A. Oral B. IM C. IV D. Aerosol Answer: D Explanation: (A) Ribavirin is not supplied in an oral form. (B) Ribavirin is administered by aerosol in order to decrease the duration of viral shedding within the infected tissue. (C) Ribavirin is not approved for IV use to treat respiratory syncytial virus. (D) Ribavirin is a synthetic antiviral agent supplied as a crystalline powder that is reconstituted with sterile water. A Small Aerosol Particle Generator unit aerosolizes the medication for delivery by oxygen hood, croup tent, or aerosol mask. QUESTION 167 A pregnant client is at the clinic for a third trimester prenatal visit. During this examination, it has been NCLEX NCLEX-RN determined that her fetus is in a vertex presentation with the occiput located in her right anterior quadrant. On her chart this would be noted as: A. Right occipitoposterior B. Right occipitoanterior C. Right sacroanterior D. LOA Answer: B Explanation: (A) The fetus in the right occipitoposterior position would be presenting with the occiput in the maternal right posterior quadrant. (B) Fetal position is defined by the location of the fetal presenting part in the four quadrants of the maternal pelvis. The right occipitoanterior is a fetus presenting with the occiput in mother‘s right anterior quadrant. (C) The fetus in right sacroanterior position would be presenting a sacrum, not an occiput. (D) The fetus in left occipitoanterior position would be presenting with the occiput in the mother‘s left anterior quadrant. QUESTION 168 Which of the following menu choices would indicate that a client with pressure ulcers understands the role diet plays in restoring her albumin levels? A. Broiled fish with rice B. Bran flakes with fresh peaches C. Lasagna with garlic bread D. Cauliflower and lettuce salad Answer: A Explanation: (A) Broiled fish and rice are both excellent sources of protein. (B) Fresh fruits are not a good source of protein. (C) Foods in the bread group are not high in protein. (D) Most vegetables are not high in protein; peas and beans are the major vegetables higher in protein. QUESTION 169 A 42-year-old client with bipolar disorder has been hospitalized on the inpatient psychiatric unit. She is dancing around, talking incessantly, and singing. Much of the time the client is anorexic and eats very little from her tray before she is up and about again. The nurse‘s intervention would be to: A. Confront the client with the fact that she will have to eat more from her tray to sustain her B. Try to get the client to focus on her eating by offering a detailed discussion on the importance of nutrition C. Let her have snacks and drinks anytime that she wants them because she will not eat at regular meal times D. Not expect the client to sit down for complete meals; monitor intake, offering snacks and juice frequently Answer: D Explanation: A. The manic client‘s mood may easily change from euphoric to irritable. The nurse should avoid confrontation and let the client know what she can do, rather than what she cannot. B. Although helpful to refocus or redirect the manic client to discuss only one topic at a time, distractibility is NCLEX NCLEX-RN Lamazeprepared laboring woman is taught to breathe with her chest, not abdominal, muscles. (D) When deep chest breathing patterns are used in Lamaze preparation, they are slowly paced at a rate of 6–9 breaths/min. QUESTION 175 A couple is planning the conception of their first child. The wife, whose normal menstrual cycle is 34 days in length, correctly identifies the time that she is most likely to ovulate if she states that ovulation should occur on day: A. 14+2 days B. 16+2 days C. 20+2 days D. 22+2 days Answer: C Explanation: (A) Ovulation is dependent on average length of menstrual cycle, not standard 14 days. (B) Ovulation occurs 14+2 days before next menses (34 minus 14 does not equal 16). (C) Ovulation occurs 14+2 days before next menses (34 minus 14 equals 20). (D) Ovulation occurs 14+2 days before next menses (34 minus 14 does not equal 22). QUESTION 176 A nurse should carefully monitor a client for the following side effect of MgSO4: A. Visual blurring B. Tachypnea C. Epigastric pain D. Respiratory depression Answer: D Explanation: (A, C) The nurse should provide good distractors because these symptoms indicate that PIH has become more severe and may precede the convulsive or eclamptic phase. (B) This is the oppositeside effect of this medication. (D) This is a common side effect of this medication and needs to be monitored and recorded frequently. QUESTION 177 The nurse would teach a male client ways to minimize the risk of infection after eye surgery. Which of the following indicates the client needs further teaching? A. ―I will wash my hands before instilling eye medications.‖ B. ―I will wear sunglasses when going outside.‖ C. ―I will wear an eye patch for the first 3 postoperative days.‖ D. ―I will maintain the sterility of the eye medications.‖ Answer: C Explanation: (A) Hand washing would be an important action designed to prevent transmission of pathogens from the hands to the eye. (B) Wearing sunglasses when going outside will prevent airborne NCLEX NCLEX-RN pathogens from entering the eye. (C) Eye patches are most frequently ordered to be worn while the client sleeps or naps, not constantly for this length of time. (D) Eye medications are sterile; clients need to be taught how to maintain this sterility. QUESTION 178 With a geriatric client, the nurse should also assess whether he has been obtaining a yearly vaccination against influenz A. Why is this assessment important? A. Influenza is growing in our society. B. Older clients generally are sicker than others when stricken with flu. C. Older clients have less effective immune systems. D. Older clients have more exposure to the causative agents. Answer: C Explanation: (A) Although influenza is common, the elderly are more at risk because of decreased effectiveness of their immune system, not because the incidence is increasing. (B) Older clients have the same degree of illness when stricken as other populations. (C) As people age, their immune system becomes less effective, increasing their risk for influenz A. (D) Older clients have no more exposure to the causative agents than do school-age children, for example. QUESTION 179 A client delivered her first-born son 4 hours ago. She asks the nurse what the white cheeselike substance is under the baby‘s arms. The nurse should respond: A. ―This is a normal skin variation in newborns. It will go away in a few days.‖ B. ―Let me have a closer look at it. The baby may have an infection.‖ C. ―This material, called vernix, covered the baby before it was born. It will disappear in a few days.‖ D. ―Babies sometimes have sebaceous glands that get plugged at birth. This substance is an example of that condition.‖ Answer: C Explanation: (A) This response identifies the fact that vernix is a normal neonatal variation, but it does not teach the client medical terms that may be useful in understanding other healthcare personnel. (B) This response may raise maternal anxiety and incorrectly identifies a normal neonatal variation. (C) This response correctly identifies this neonatal variation and helps the client to understand medical terms as well as the characteristics of her newborn. (D) Blocked sebaceous glands produce milia, particularly present on the nose. QUESTION 180 A 26-year-old client is in a treatment center for aprazolam (Xanax) abuse and continues to manifest moderate levels of anxiety 3 weeks into the rehabilitation program, often requesting medication for ―his nerves.‖ Included in the client‘s plan of care is to identify alternate methods of coping with stress and anxiety other than use of medication. After intervening with assistance in stress reduction techniques, identifying feelings and past coping, the nurse evaluates the outcome as being met if: A. Client promises that he will not abuse aprazolam after discharge NCLEX NCLEX-RN B. Client demonstrates use of exercise or physical activity to handle nervous energy following conflicts of everyday life C. Client is able to verbalize effects of substance abuse on the body D. Client has remained substance free during hospitalization and is discharged Answer: B Explanation: (A) This client response does not address stress reduction techniques. Verbal response focuses only on the problem. (B) Exercise or physical activity is a common strategy or coping technique used to reduce stress and anxiety. (C) Verbalizing effects of substance abuse on the body may help with insight and break through denial, but it is not a strategy to reduce anxiety. (D)Remaining substance-free does indicate motivation to change lifestyle of substance abuse or dependence, and it is not a stress reduction strategy in itself. QUESTION 181 In assessing the nature of the stool of a client who has cystic fibrosis, what would the nurse expect to see? A. Clay-colored stools B. Steatorrhea stools C. Dark brown stools D. Blood-tinged stools Answer: B Explanation: (A) Clay-colored stools indicate dysfunction of the liver or biliary tract. (B) In the early stages of cystic fibrosis, fat absorption is primarily affected resulting in fat, foul, frothy, bulky stools. (C) Dark brown stools indicate normal passage through the colon. (D) Blood-tinged stools indicate dysfunction of the gastrointestinal (GI) tract. QUESTION 182 A client is taught to eat foods high in potassium. Which food choices would indicate that this teaching has been successful? A. Pork chop, baked acorn squash, brussel sprouts B. Chicken breast, rice, and green beans C. Roast beef, baked potato, and diced carrots D. Tuna casserole, noodles, and spinach Answer: A Explanation: (A) Both acorn squash and brussels sprouts are potassium-rich foods. (B) None of these foods is considered potassium rich. (C) Only the baked potato is a potassium-rich food. (D) Spinach is the only potassium-rich food in this option. QUESTION 183 The nurse is teaching a 10-year-old insulin-dependent diabetic how to administer insulin. Which one of the following steps must be taught for insulin administration? A. Never use abdominal site for a rotation site. B. Pinch the skin up to form a subcutaneous pocket. NCLEX NCLEX-RN used to assess the home environment in areas of social, emotional, and cognitive supports. NCLEX NCLEX-RN QUESTION 189 A gravida 2 para 1 client is hospitalized with severe preeclampsi A. While she receives magnesium sulfate (MgSO4) therapy, the nurse knows it is safe to repeat the dosage if: A. Deep tendon reflexes are absent B. Urine output is 20 mL/hr C. MgSO4serum levels are>15 mg/dL D. Respirations are>16 breaths/min Answer: D Explanation: (A) MgSO4is a central nervous system depressant. Loss of reflexes is often the first sign of developing toxicity. (B) Urinary output at <25 mL/hr or 100 mL in 4 hours may result in the accumulation of toxic levels of magnesium. (C) The therapeutic serum range for MgSO4is 6–8 mg/dL. Higher levels indicate toxicity. (D) Respirations of>16 breaths/min indicate that toxic levels of magnesium have not been reached. Medication administration would be safe. QUESTION 190 In addition to changing the mother‘s position to relieve cord pressure, the nurse may employ the following measure (s) in the event that she observes the cord out of the vagina: A. Immediately pour sterile saline on the cord, and repeat this every 15 minutes to prevent drying. B. Cover the cord with a wet sponge. C. Apply a cord clamp to the exposed cord, and cover with a sterile towel. D. Keep the cord warm and moist by continuous applications of warm, sterile saline compresses. Answer: D Explanation: (A) Saline should be warmed; waiting 15 minutes may not keep the cord moist. (B) This choice does not specify what the sponge was ―wet‖ with. (C) This measure would stop circulation to the fetus. (D) The cord should be kept warm and moist to maintain fetal circulation. This measure is an accepted nursing action. QUESTION 191 Following a gastric resection, which of the following actions would the nurse reinforce with the client in order to alleviate the distress from dumping syndrome? A. Eating three large meals a day B. Drinking small amounts of liquids with meals C. Taking a long walk after meals D. Eating a low-carbohydrate diet Answer: D Explanation: (A) Six small meals are recommended. (B) Liquids after meals increase the time food empties from the stomach. (C) Lying down after meals is recommended to prevent gravity from producing dumping. (D) A low- NCLEX NCLEX-RN carbohydrate diet will prevent a hypertonic bolus, which causes dumping. NCLEX NCLEX-RN food choices indicate that this teaching has been understood? A. Omelette and hash browns B. Pancakes and syrup C. Bagel with cream cheese D. Cooked oatmeal and grapefruit half Answer: D Explanation: (A) Eggs and hash browns do not provide much fiber and bulk, so they do not effectively prevent constipation. (B) Pancakes and syrup also have little fiber and bulk, so they do not effectively prevent constipation. (C) Bagel and cream cheese do not provide intestinal bulk. (D) A combination of oatmeal and fresh fruit will provide fiber and intestinal bulk. QUESTION 198 In cleansing the perineal area around the site of catheter insertion, the nurse would: A. Wipe the catheter toward the urinary meatus B. Wipe the catheter away from the urinary meatus C. Apply a small amount of talcum powder after drying the perineal area D. Gently insert the catheter another 12 inch after cleansing to prevent irritation from the balloon Answer: B Explanation: (A) Wiping toward the urinary meatus would transport microorganisms from the external tubing to the urethra, thereby increasing the risk of bladder infection. (B) Wiping away from the urinary meatus would remove microorganisms from the point of insertion of the catheter, thereby decreasing the risk of bladder infection. (C) Talcum powder should not be applied following catheter care, because powders contribute to moisture retention and infection likelihood. (D) The catheter should never be inserted further into the urethra, because this would serve no useful purpose and would increase the risk of infection. QUESTION 199 A 7-year-old child is brought to the ER at midnight by his mother after symptoms appeared abruptly. The nurse‘s initial assessment reveals a temperature of 104.5F (40.3C), difficulty swallowing, drooling, absence of a spontaneous cough, and agitation. These symptoms are indicative of which one of the following? A. Acute tracheitis B. Acute spasmodic croup C. Acute epiglottis D. Acute laryngotracheobronchitis Answer: C Explanation: (A) Clinical manifestations of acute tracheitis include a 2–3 day history of URI, croupy cough, stridor, purulent secretions, high fever. (B) Clinical manifestations of spasmodic croup include a history of URI, croupy cough, stridor, dyspnea, low-grade fever, and a slow progression. The age group most affected is 3 months to 3 years. (C) Three clinical observations have been found to be predictive of epiglottitis: the presence of drooling, absence of spontaneous cough, and agitation.Epiglottitis has a rapid onset that is accompanied by high fever and NCLEX NCLEX-RN dysphagi A. (D) Clinical manifestations of acute laryngotracheobronchitis (LTB) include slow onset with a history of URI, low-grade fever, stridor, brassy cough, and irritability. QUESTION 200 A client tells the nurse that she has had a history of urinary tract infections. The nurse would do further health teaching if she verbalizes she will: A. Drink at least 8 oz of cranberry juice daily B. Maintain a fluid intake of at least 2000 mL daily C. Wash her hands before and after voiding D. Limit her fluid intake after 6 PM so that there is not a great deal of urine in her bladder while she sleeps Answer: D Explanation: (A) Cranberry juice helps to maintain urine acidity, thereby retarding bacterial growth. (B) A generous fluid intake will help to irrigate the bladder and to prevent bacterial growth within the bladder. (C) Hand washing is an effective means of preventing pathogen transmission. (D) Restricting fluid intake would contribute to urinary stasis, which in turn would contribute to bacterial growth. QUESTION 201 A pregnant client experiences a precipitous delivery. The nursing action during a precipitous delivery is to: A. Control the delivery by guiding expulsion of fetus B. Leave the room to call the physician C. Push against the perineum to stop delivery D. Cross client‘s legs tightly Answer: A Explanation: (A) Controlling the rapid delivery will reduce the risk of fetal injury and perineal lacerations. (B) The nurse should always remain with a client experiencing a precipitous delivery. (C) Pushing against the perineum may cause fetal distress. (D) Crossing of legs may cause fetal distress and does not stop the delivery process. QUESTION 202 A client is diagnosed with organic brain disorder. The nursing care should include: A. Organized, safe environment B. Long, extended family visits C. Detailed explanations of procedures D. Challenging educational programs Answer: A Explanation: (A) A priority nursing goal is attending to the client‘s safety and well-being. Reorient frequently, remove dangerous objects, and maintain consistent environment. (B) Short, NCLEX NCLEX-RN frequent visits are recommended to avoid overstimulation and fatigue. (C) Short, concise, simple explanations are easier to understand. (D) Mental capability and attention span deficits make learning difficult and frustrating. QUESTION 203 A premature infant needs supplemental O2 therapy. A nursing intervention that reduces the risk of retrolental fibroplasia is to: A. Maintain O2at <40% B. Maintain O2at>40% C. Give moist O2at>40% D. Maintain on 100% O2 Answer: A Explanation: (A) Retrolental fibroplasia is the result of prolonged exposure to high levels of O2in premature infants. Complications are hemorrhage and retinal detachment. (B, C, D) O2concentration is too high. QUESTION 204 A 35-year-old client is admitted to the hospital with diabetic ketoacidosis. Results of arterial blood gases are pH 7.2, PaO2 90, PaCO2 45, and HCO3 16. The nursing assessment of arterial blood gases indicate the presence of: A. Respiratory alkalosis B. Respiratory acidosis C. Metabolic alkalosis D. Metabolic acidosis Answer: D Explanation: (A) Respiratory alkalosis is determined by elevated pH and low PaCO2. (B) Respiratory acidosis is determined by low pH and elevated PaCO2. (C) Metabolic alkalosis is determined by elevated pH and HCO3.(D) Metabolic acidosis is determined by low pH and HCO3. QUESTION 205 A client presents to the psychiatric unit crying hysterically. She is diagnosed with severe anxiety disorder. The first nursing action is to: A. Demand that she relax B. Ask what is the problem C. Stand or sit next to her D. Give her something to do Answer: C Explanation: (A) This nursing action is too controlling and authoritative. It could increase the client‘s anxiety level. (B) In her anxiety state, the client cannot rationally identify a problem. (C) This nursing action conveys a message of caring and security. (D) Giving the client a task would increase her anxiety. This would be a late nursing action. NCLEX NCLEX-RN because it: A. Prevents administration of other drugs B. Prevents entry of air into tubing C. Prevents inadvertent administration of a large amount of fluids D. Prevents phlebitis Answer: C Explanation: (A) A volume control set has a chamber that permits the administration of compatible drugs. (B) Air may enter a volume control set when tubing is not adequately purged. (C) A volume control set allows the nurse to control the amount of fluid administered over a set period. (D) Contamination of volume control set may cause phlebitis. QUESTION 212 A pregnant client experiences spontaneous rupture of membranes. The first nursing action is to: A. Assess the client‘s respirations B. Notify the physician C. Auscultate fetal heart rate D. Transfer to delivery suite Answer: C Explanation: (A) Immediately following membrane rupture, the fetus is at risk for complications, not necessarily the mother. (B) The physician is notified after the nurse completes an assessment of the mother‘s and fetus‘s conditions. (C) Rupture of membranes facilitates fetal descent. A potential complication is cord prolapse, which is assessed by auscultating fetal heart rate. (D) Rupture of membranes does not necessarily indicate readiness to deliver. QUESTION 213 A 30-year-old client has a history of several recent traumatic experiences. She presents at the physician‘s office with a complaint of blindness. Physical exam and diagnostic testing reveal no organic cause. The nurse recognizes this as: A. Delusion B. Illusion C. Hallucination D. Conversion Answer: D Explanation: (A) The client‘s blindness is real. Delusion is a false belief. (B) Illusion is the misrepresentation of a real, external sensory experience. (C) Hallucination is a false sensory perception involving any of the senses. (D) Conversion is the expression of intrapsychic conflict through sensory or motor manifestations. QUESTION 214 A client was not using his seat belt when involved in a car accident. He fractured ribs 5, 6, and 7 on the left and developed a left pneumothorax. Assessment findings include: NCLEX NCLEX-RN A. Crackles and paradoxical chest wall movement B. Decreased breath sounds on the left and chest pain with movement C. Rhonchi and frothy sputum D. Wheezing and dry cough Answer: B Explanation: (A) Crackles are caused by air moving through moisture in the small airways and occur with pulmonary edem A. Paradoxical chest wall movement occurs with flail chest when a segment of the thorax moves outward on inspiration and inward on expiration. (B) Decreased breath sounds occur when a lung is collapsed or partially collapsed. Chest pain with movement occurs with rib fractures. (C) Rhonchi are caused by air moving through large fluid-filled airways. Frothy sputum may occur with pulmonary edem A. (D) Wheezing is caused by fluid in large airways already narrowed by mucus or bronchospasm. Dry cough could indicate a cardiac problem. QUESTION 215 An 11-year-old boy has received a partial-thickness burn to both legs. He presents to the emergency room approximately 15 minutes after the accident in excruciating pain with charred clothing to both legs. What is the first nursing action? A. Apply ice packs to both legs. B. Begin débridement by removing all charred clothing from wound. C. Apply Silvadene cream (silver sulfadiazine). D. Immerse both legs in cool water. Answer: D Explanation: (A) Ice creates a dramatic temperature change in the tissue, which can cause further thermal injury. (B) Charred clothing should not be removed from wound first. This creates further tissue damage. Débridement is not the first nursing action. (C) Applying silver sulfadiazine cream first insulates heat in injured tissue and increases potential for infection. (D) Emergency care of a thermal burn is immersing both legs in cool water. Cool water permits gradual temperature change and prevents further thermal damage. QUESTION 216 The physician is preparing to induce labor on a 40-week multigravid A. The nurse should anticipate the administration of: A. Oxytocin (Pitocin) B. Progesterone C. Vasopressin (Pitressin) D. Ergonovine maleate Answer: A Explanation: (A) Oxytocin is a hormone secreted by the neurohypophysis during suckling and parturition that produces NCLEX NCLEX-RN strong uterine contractions. (B) Progesterone has a quiescence effect on the uterus. (C) Vasopressin is an antidiuretic hormone that promotes water reabsorption by the renal tubules. (D) Ergonovine produces dystocia as a result of sustained uterine contractions. QUESTION 217 A pregnant client during labor is irritable and feels the urge to vomit. The nurse should recognize this as the: A. Fourth stage of labor B. Third stage of labor C. Transition stage of labor D. Second stage of labor Answer: C Explanation: (A) The fourth stage begins after expulsion of the placent A. Client symptoms are: fatigue; chills; scant, bloody vaginal discharge; and nause A. (B) The third stage is from birth to expulsion of placent A. Client symptoms are uterine contractions, gush of blood, and perineal pain. (C) The transition stage is characterized by strong uterine contractions and cervical dilation. Clientsymptoms are irritability, restlessness, belching, muscle tremors, nausea, and vomiting. (D) The second stage is characterized by full dilation of cervix. Client symptoms are perineal bulge, pushing with contractions, great irritability, and leg cramps. QUESTION 218 A physician‘s order reads: Administer KCl 10% oral solution 1.5 mL. The KCl bottle reads 20 mEq/15 mL. What dosage should the nurse administer to the infant? A. 1 mEq B. 1.13 mEq C. 2 mEq D. Not enough information to calculate Answer: C Explanation: (A) This answer is a miscalculation. (B) This answer is a miscalculation. (C) 1.33 mEq = 1 mL, then 1.5 mL X=1.99, or 2 mEq. (D) Information is adequate for calculation. QUESTION 219 At 38 weeks‘ gestation, a client is in active labor. She is using her Lamaze breathing techniques. The RN is coaching her breathing and encouraging her to relax and work with her contractions. Which one of the following complaints by the client will alert the RN that she is beginning to hyperventilate with her breathing? A. ―I am cold.‖ B. ―I have a backache.‖ C. ―I feel dizzy.‖ D. ―I am nauseous.‖ Answer: C Explanation: NCLEX NCLEX-RN separates from the uterine wall as the uterus contracts and cervix dilates. This separation causes painless bleeding in the 7th-8th month. (D) Polyhydramnios is excessive amniotic fluid. QUESTION 225 A 66-year-old female client has smoked 2 packs of cigarettes per day for 20 years. Her arterial blood gases on room air are as follows: pH 7.35; PO2 70 mm Hg; PCO2 55 mm Hg; HCO3 32 mEq/L. These blood gases reflect: A. Compensated metabolic acidosis B. Compensated respiratory acidosis C. Compensated respiratory alkalosis D. Uncompensated respiratory acidosis Answer: B Explanation: (A) In compensated metabolic acidosis, the pH level is normal, the PCO2level is decreased, and the HCO3level is decreased. The client‘s primary alteration is an inability to remove excess acid via the kidneys. The lungs compensate by hyperventilating and decreasing PCO2. (B) In compensated respiratory acidosis, the pH level is normal, the PCO2level is elevated, and the HCO3level is elevated. The client‘s primary alteration is an inability to remove CO2from the lungs, so over time, the kidneys increase reabsorption of HCO3to buffer the CO2. (C) In compensated respiratory alkalosis, the pH level is normal, the PCO2level is decreased, and the HCO3level is decreased. The client‘s primary alteration is hyperventilation, which decreases PCO2. The client compensates by increasing the excretion of HCO3from the body. (D) In uncompensated respiratory acidosis, the pH level is decreased, the PCO2level is increased, and the HCO3level is normal. The client‘s primary alteration is an inability to remove CO2from the lungs. The kidneys have not compensated by increasing HCO3reabsorption. QUESTION 226 The client has been in active labor for the last 12 hours. During the last 3 hours, labor has been augmented with oxytocin because of hypoactive uterine contractions. Her physician assesses her cervix as 95% effaced, 8 cm dilated, and the fetus is at 0 station. Her oral temperature is 100.2F at this time. The physician orders that she be prepared for a cesarean delivery. In preparing the client for the cesarean delivery, which one of the following physician‘s orders should the RN question? A. Administer meperidine (Demerol) 100 mg IM 1 hour prior to the delivery. B. Discontinue the oxytocin infusion. C. Insert an indwelling Foley catheter prior to delivery. D. Prepare abdominal area from below the nipples to below the symphysis pubis are A. Answer: A Explanation: (A) Meperidine is a narcotic analgesic medication that crosses the placental barrier and reaches the fetus, causing respiratory depression in the fetus. A narcotic medication should never be included in the preoperative order for a cesarean delivery. (B) Oxytocin infusion would be discontinued if client is being prepared for a cesarean delivery because the medication would not be needed. (C) The bladder is always emptied prior to and during the surgical intervention to prevent the urinary bladder from accidentally being incised while the uterine incision is made. (D) The abdominal area is always prepared to rid the area of hair before the abdominal NCLEX NCLEX-RN incision is made. Abdominal hair cannot be sterilized and could become a source for postoperative incisional infection. QUESTION 227 A 17-year-old client has a T-4 spinal cord injury. At present, he is learning to catheterize himself. When he says, ―This is too much trouble. I would rather just have a Foley.‘‘ An appropriate response for the RN teaching him would be: A. ―I know. It is a lot to learn. In the long run, though, you will be able to reduce infections if you do an intermittent catheterization program.‘‘ B. ―It is not too much trouble. This is the best way to manage urination.‘‘ C. ―OK. I‘ll ask your physician if we can replace the Foley.‘‘ D. ―You need to learn this because your doctor ordered it.‘‘ Answer: A Explanation: (A) This response acknowledges the client‘s feelings, gives him factual information, and acknowledges that the final decision is his. (B) This response is judgmental and discourages the client from expressing his feelings about the procedure. (C) Catheterization is a procedure thattakes time to learn, but which, for the spinal cord– injured client, can significantly reduce the incidence of urinary tract infections. A young client with a T-4 injury has the hand function to learn this procedure fairly easily. (D) The final decision about bladder elimination management ultimately rests with the client and not the physician. QUESTION 228 A cardinal symptom of the schizophrenic client is hallucinations. A nurse identifies this as a problem in the category of: A. Impaired communication B. Sensory-perceptual alterations C. Altered thought processes D. Impaired social interaction Answer: B Explanation: (A) Impaired communication refers to decreased ability or inability to use or understand language in an interaction. (B) In sensory-perceptual alterations an individual has distorted, impaired, or exaggerated responses to incoming stimuli (i.e., a hallucination, which is a false sensory perception that is not associated with real external stimuli). (C) An altered thought processes problem statement is used when an individual experiences a disruption in cognitive operations and activities (i.e., delusions, loose associations, ideas of reference). (D) In impaired social interaction, the individual participates too little or too much in social interactions. QUESTION 229 A client calls the prenatal clinic to schedule an appointment. She states she has missed three menstrual periods and thinks she might be pregnant. During her first visit to the prenatal clinic, it is confirmed that she is pregnant. The registered nurse (RN) learns that her last menstrual period began on June 10. According to Nägele‘s rule, the estimated date of confinement is: NCLEX NCLEX-RN A. March 17 B. June 3 C. August 30 D. January 10 Answer: A Explanation: (A) Using Nägele‘s rule, count back 3 calendar months from the first day of the last menstrual period. The answer is March 10. Then add 7 days and 1 year, which would be March 17 of the following year. (B, C, D) This date is incorrect. QUESTION 230 A 55-year-old client is unconscious, and his physician has decided to begin tube feeding him using a smallbore silicone feeding tube (Keofeed, Duo-Tube). After the tube is inserted, the nurse identifies the most reliable way to confirm appropriate placement is to: A. Aspirate gastric contents B. Auscultate air insufflated through the tube C. Obtain a chest x-ray D. Place the tip of the tube under water and observe for air bubbles Answer: C Explanation: (A) Aspiration of gastric contents is usually a reliable way to verify tube placement. However, if the client has dark respiratory secretions from bleeding, tube feedings could be mistaken for respiratory secretions; in other words, aspirating an empty stomach is less reliable in this instance. In addition, it is common for small-bore feeding tubes to collapse when suction pressure is applied. (B) Insufflation of air into large-bore nasogastric tubes can usually be clearly heard. In small-bore tubes, it is more difficult to hear air, and it is difficult to distinguish between air in the stomach and air in the esophagus. (C) A chest x-ray is the most reliable means to determine placement of small-bore nasogastric tubes. (D) Observing for air bubbles when the tip is held under water is an unreliable means to determine correct tube placement for all types of nasogastric tubes. Air may come from both the respiratory tract and the stomach, and the client who is breathing shallowly may not force air out of the tube into the water. QUESTION 231 A client had a vaginal delivery 3 days ago and is discharged from the hospital on the 2nd day postpartum. She told the RN, ―I need to start exercising so that I can get back into shape. Could you suggest an exercise I could begin with?‘‘ The RN could suggest which one of the following? A. Push-ups B. Jumping jacks C. Leg lifts D. Kegel exercises Answer: D Explanation: (A, B, C) This exercise is too strenuous at this time. (D) This exercise is recommended for the first few days NCLEX NCLEX-RN Answer: B Explanation: (A) Increased calories may result in weight gain, but there is no indication in this question that this man‘s diet has changed in a way that would result in increased calories. (B) Decreasing cardiac output stimulates the renin- angiotensin-aldosterone cycle and results in fluid retention, which is reflected by weight gain. (C) Decreasing renal function may result in fluid retention, but this question gives no indication that this man has any renal problems. (D) Profound diuresis occurs with diabetes insipidus, which results in weight loss. QUESTION 238 The nurse is assessing and getting a history from a client treated for depression with a monoamine oxidase (MAO) antidepressant. The most serious side effect associated with this antidepressant and the ingestion of tyramine in aged foods may be: A. Hypertensive crisis B. Severe rash C. Severe hypotension D. Severe diarrhea Answer: A Explanation: (A) The most serious adverse reactions of MAO inhibitors involve blood pressure and ingestion of tyramine- containing foods, which may provoke a hypertensive crisis. (B) MAO inhibitors cause adverse reactions affecting the central nervous system and serious adverse reactions involving blood pressure. (C) MAO inhibits false neurotransmitters (phenylalanines) and may produce hypotensive reactions from gradual accumulation of these neurotransmitters. (D) The most serious adverse reactions of MAO inhibitors involve blood pressure. QUESTION 239 When planning care for the passive-aggressive client, the nurse includes the following goal: A. Allow the client to use humor, because this may be the only way this client can express self. B. Allow the client to express anger by using ―I‖ messages, such as ―I was angry when . . .,‖ etc. C. Allow the client to have time away from therapeutic responsibilities. D. Allow the client to give excuses if he forgets to give staff information. Answer: B Explanation: (A) Ceasing to use humor and sarcasm is a more appropriate goal, because this client uses these behaviors covertly to express aggression instead of being open with anger. (B) Use of ―I‖ messages demonstrates proper use of assertive behavior to express anger instead of passive-aggressive behavior. (C) Client is expected to complete share of work in therapeutic community because he has often obstructed other‘s efforts by failing to do his share. (D) Client has used conveniently forgetting or withholding information as a passive-aggressive behavior, which is not acceptable. QUESTION 240 A newborn is admitted to the newborn nursery with tremors, apnea periods, and poor sucking reflex. The nurse should suspect: NCLEX NCLEX-RN A. Central nervous system damage B. Hypoglycemia C. Hyperglycemia D. These are normal newborn responses to extrauterine life Answer: B Explanation: (A) Central nervous system damage presents as seizures, decreased arousal, and absence of newborn reflexes. (B) In a diabetic mother, the infant is exposed to high serum glucose. The fetal pancreas produces large amounts of insulin, which causes hypoglycemia after birth. (C) Hypoglycemia is a common newborn problem. Increased insulin production causes hypoglycemia, not hyperglycemi A. (D) These are not normal adaptive behaviors to extrauterine life. QUESTION 241 A schizophrenic client who is experiencing thoughts of having special powers states that ―I am a messenger from another planet and can rule the earth.‖ The nurse assesses this behavior as: A. Ideas of reference B. Delusions of persecution C. Thought broadcasting D. Delusions of grandeur Answer: D Explanation: (A) Clients experiencing ideas of reference believe that information from the environment (e.g., the television) is referring to them. (B) Clients experiencing delusions of persecution believe that others in the environment are plotting against them. (C) Clients experiencing thought broadcasting perceive that others can hear their thoughts. (D) Clients experiencing delusions of grandeur think that they are omnipotent and have superhuman powers. QUESTION 242 A newborn infant is exhibiting signs of respiratory distress. Which of the following would the nurse recognize as the earliest clinical sign of respiratory distress? A. Cyanosis B. Increased respirations C. Sternal and subcostal retractions D. Decreased respirations Answer: C Explanation: (A) Cyanosis is a late clinical sign of respiratory distress. (B) Rapid respirations are normal in a newborn. (C) The newborn has to exert an extra effort for ventilation, which is accomplished by using the accessory muscles of ventilation. The diaphragm and abdominal muscles are immature and weak in the newborn. (D) Decreased respirations are a late clinical sign. In the newborn, decreased respirations precede respiratory failure. QUESTION 243 After the RN is finished the initial assessment of a newborn baby and after the initial bonding between the NCLEX NCLEX-RN newborn and the mother has taken place in the delivery room, the RN will bring the newborn to the well-baby nursery. Before the newborn is taken from the delivery room and brought to the well-baby nursery, the RN makes sure that which of the following interventions was completed? A. The physician verifies the exact time of birth. B. The nurse counts the instruments and sponges with the scrub nurse. C. The nurse instills prophylactic ointment in the conjunctival sacs of the newborn‘s eyes. D. The nurse makes sure the mother and her newborn have been tagged with identical bands. Answer: D Explanation: (A) The delivery room personnel are responsible for verifying time of birth. (B) The scrub and circulating nurses count sponges and instruments. (C) This intervention is done in the nursery. (D) Tagging the mother and infant with identical bands is of utmost importance. The mother wears one band, and the newborn wears two. Identical numbers on the three bands provide identification for the newborn and the birth mother. Every time the newborn is brought to the mother after delivery, those bands are checked to be sure that the numbers are identical. QUESTION 244 A client experiencing delusions states, ―I came here because there were people surrounding my house that wanted to take me away and use my body for science.‖ The best response by the nurse would be: A. ―Describe the people surrounding your house that want to take you away.‖ B. ―I need more information on why you think others want to use your body for science.‖ C. ―There were no people surrounding your house, your relatives brought you here, and no one really wants your body for science.‖ D. ―I know that must be frightening for you; let the staff know when you are having thoughts that trouble you.‖ Answer: D Explanation: (A) Focusing on the delusional content does not reinforce reality. (B) Pursuing details or more information on the delusion reinforces the false belief and further distances the client from reality. (C) Challenging the client‘s delusional system may force the client to defend it, and you cannot change the delusion through logic. (D) Focusing on the feeling can reinforce reality and discourage the false belief. Seeking out staff when thoughts are troublesome can help to decrease anxiety. QUESTION 245 On admission, the client has signs and symptoms of pulmonary edem A. The nurse places the client in the most appropriate position for a client in pulmonary edema, which is: A. High Fowler B. Lying on the left side C. Sitting in a chair D. Supine with feet elevated Answer: A Explanation: (A) High Fowler position decreases venous return to the heart and permits greater lung expansion so that
Docsity logo



Copyright © 2024 Ladybird Srl - Via Leonardo da Vinci 16, 10126, Torino, Italy - VAT 10816460017 - All rights reserved